Exam Master - EENT
A 30-year-old Black male patient is admitted to the hospital to undergo stapedectomy for the treatment of otosclerosis. He has experienced increased hearing loss in the right ear for a few years. The patient's mother had had the same condition when she was in her 40s, and she underwent a successful operation. Assessment is performed using the Weber and Rinne tests. Question: What physical exam finding is expected? - Bone conduction of the affected side is greater than air conduction -Bone conduction of the affected side is equal to air conduction. -Air conduction of the affected side is greater than bone conduction. -Sound lateralizes to the unaffected ear. -Bone conduction of the unaffected ear is greater than air conduction
- Bone conduction of the affected side is greater than air conduction *Otosclerosis is a pathological condition of the middle ear where there is a formation of spongy bone near the footplate of the stapes. As it advances, it causes progressive fixation of the stapes footplate. Otosclerosis typically causes a conductive hearing loss, so the sound transmission by air conduction from the stapes via the oval window to the perilymph of the inner ear is reduced. Weber tuning fork test is performed by placing the stem of a vibrating tuning fork on the midline of the head and having the patient indicate which ear hears the tone. The fork stimulates each inner ear equally. A patient with a unilateral conductive hearing loss hears the tone louder in the affected ear and a patient with a sensorineural loss hears the tone louder in the unaffected ear. In otosclerosis, the sound lateralizes to the affected ear. The Rinne tuning fork test compares air to bone conduction. The stem of the vibrating tuning fork is placed on the mastoid process, the tines of the tuning fork are held in front of the ear, and the patient is asked which stimulus is perceived better. In a healthy patient, the tone is heard longer and louder by air conduction; with a conductive hearing loss, it is reversed. In otosclerosis, the bone conduction is better than air conduction (BC>AC). With a sensorineural hearing loss, both receptions are reduced, but in the same ratio.
A 41-year-old man presents for evaluation of hearing loss. He states that he is having more difficulty in his right ear than his left. He began to notice this about 6 months ago when, while talking on his cell phone, he had to routinely switch to his left ear because of difficulty understanding the words while listening with his right ear. He states that he has had ear drainage for approximately 6 months. He also states that he was in the Navy for a few years and took up scuba diving as a recreational activity. He recalls multiple ear infections during his time in the Navy. Question - During the otoscope examination, you note deep retraction pockets, a white mass behind the tympanic membrane, and focal granulation at the peripheral of the tympanic membrane. What is the most definitive treatment for this patient? Answer Choices - 10 days of therapy with second-generation cephalosporin - 14 days of therapy with an otic solution of antimicrobials and corticosteroids - Close observation and followup in 2 weeks - Close observation and referral to an otologist for custom-made earplugs - Referral to an otolaryngologist for surgical intervention
- Referral to an otolaryngologist for surgical intervention Explanation - The treatment for cholesteatoma is surgical removal or marsupialization of the sac. This should be performed by a specialist. Antibiotics and/or steroids can be tried as nonsurgical measures, but the question asks for the definitive treatment. Observation and follow-up or earplugs are not recommended since the cholesteatoma may cause destruction of the middle ear ossicles and spread to the mastoid process, worsening the condition.
A 21-year-old man reports intermittent sneezing, runny nose, itchy nose and eyes, and cough. His symptoms typically occur after exposure to pollen or pet dander. He has a history of a fractured clavicle but is otherwise healthy. He has no known drug allergies and takes Tylenol for occasional headache. He is a full-time engineering student and denies any alcohol or drug use. Physical exam reveals an alert Caucasian male with dark circles under both eyes. Physical exam is remarkable for erythematous injection of the conjunctiva, pale boggy nasal mucosa, and postnasal drainage. Question - What is the best initial treatment for this patient? Answer Choices 1 Antihistamine 2 Decongestant 3 Oral steroids 4 Saline nasal wash 5 Topical nasal decongestant spray
1 Antihistamine -This patient has symptoms of mild and intermittent allergic rhinitis, including cough, sneezing, rhinorrhea, and erythema/itching of the eyes. The treatment of choice is second-generation non-sedating oral antihistamines or intranasal corticosteroids. Decongestants may be useful in reducing nasal congestion in allergic rhinitis, but they are not first-line therapy for the condition. Saline nasal wash may be used as a sole or complementary treatment for allergic rhinitis, but it is not the first-line choice. Topical nasal decongestant spray may be effective in reducing nasal congestion temporarily, but rebound nasal congestion may result. Use is generally limited to 3-5 days. Prolonged use of topical nasal decongestant sprays may result in a condition known as "rhinitis medicamentosa" associated with persistent nasal congestion. Oral steroids can relieve some symptoms of allergic rhinitis, but they are not first-line therapy because of the side effects of long-term use.
13-year-old girl presents with a painful reddened lesion on the inside of her cheek. She noticed this lesion a few days ago and it will not go away. Upon physical exam, the inside of her right cheek has a yellow-grey appearance with an erythematous halo surrounding the lesion. Patient overall feels well besides this specific issue, and no other rashes or lesions are noted. All laboratory findings are within normal limits. HSV-1 and HSV-2 are both negative. Question -What is the most likely diagnosis? Answer Choices 1 Aphthous ulcers 2 Folate deficiency 3 Herpangina 4 Herpes labialis 5 Lichen planus
1 Aphthous ulcers Explanation The yellow-grey appearance and erythematous halo surrounding the lesion is descriptive of aphthous ulcers. Although the patient presents with mouth sores, patients with folate deficiency present with fatigue. Herpangina presents with lesions or sores not only in the mouth, but also on the hands and feet. The lesions appear with a white base and a red border. Herpes labialis (cold sore) would involve a diagnosis of herpes simplex, and this patient's HSV-1 and HSV-2 are negative. Lichen planus presents with a lacy white patch.
A 30-year-old woman presents to the emergency room at 7 AM with severe pain and swelling of her right eye. She was awakened early the previous evening due to the discomfort and swelling of the surrounding conjunctiva. She found it difficult to sleep due to the discomfort. She planned on going to work, but the swelling had closed her eye shut, and she developed excruciating pain in the eye that radiated internally. The patient denies recent swimming and does not recollect any previous trauma or injury to the eye. She uses contact lenses, but they were not in use due to the condition of her eye. The contact lenses were stored in a small pillbox container with some fluid that she later described as tap water. She ran out of sterile cleaning and soaking solution for the contact lenses, so she has been using tap water as a substitute for approximately 5 days. She frequently sleeps with her contacts in. The patient is afebrile. Pulse is 70/min, and blood pressure is 135/80 mm Hg. Lungs are clear, and there is no evidence of lymphadenopathy. The eye has profound conjunctivitis that is acute and follicular. Purulent drainage is present. The acute nature of conjunctivitis requires an ophthalmol
1 Pseudomonas aeruginosa Pseudomonas aeruginosa causes a suppurative keratitis that is rapid in its development. It can be caused by previous injury to the eye, or by recurrent minor corneal epithelial trauma caused by contact lenses. The incidence is higher with soft lenses and extended overnight wear. Fever is usually absent, and leukocytosis is absent or minimal. The infection can lead to corneal ulceration, resulting in the rapid loss of ocular function, so these infections need to be approached as a medical emergency. Scrapings from the floor of the ulcer exhibiting gram-negative rods are strongly indicative of Pseudomonas aeruginosa and should necessitate treatment. Immediate initiation of combined topical and subconjunctival therapy with an aminoglycoside antibiotic such as gentamicin or tobramycin is advised. Topical steroids are sometimes used to reduce ocular inflammation. Pseudomonas aeruginosa is a gram-negative rod; it is a non-lactose-fermenting oxidase-positive motile bacteria. Growth on MacConkey agar is usually characterized by the production of a "grape-like" smell. A blue-green color, due to the production of the diffusible fluorescent pigments pyoverdin and pyocyanin, is characteristic of the colonies growing on MacConkey. Haemophilus aegyptius is indigenous to humans. It is an important cause of purulent conjunctivitis called "pink eye," and it can occur in outbreaks because of its contagious nature. The infection is not acute in presentation. The diffuse pink color of the sclera and the presence of a serous or purulent discharge are virtually diagnostic of Haemophilus aegyptius infection. The organism is a gram-negative coccobacillus; it is non-motile fastidious bacteria requiring the presence of special factors for its growth on agar media. Acanthamoeba is a corneal infection that occurs in healthy people and is often associated with contact lens wearers. Swimming in fresh water (where the organism is naturally found) with contact lenses can predispose the wearer to Acanthamoeba keratitis. Symptoms include blurred vision, conjunctivitis, tearing, severe pain to the eye, and photophobia. The keratitis achieves an advanced stage in several days to several months, and it can exhibit pa
A 25-year-old male patient presents with an acute otitis media with serous otitis in the right ear. Weber and Rinne tests are performed. Question: What results are expected? Answer Choices 1 Weber—sound is heard louder in right ear; Rinne—bone conduction exceeds air conduction in right ear 2 Weber—sound is heard louder in left ear; Rinne—bone conduction exceeds air conduction in right ear 3 Weber—sound is heard louder in right ear; Rinne—air conduction exceeds bone conduction in right ear 4 Weber—sound is heard louder in left ear; Rinne—air conduction exceeds air conduction in right ear 5 Weber—sound is equal in both ears; Rinne—bone conduction greater than air conduction in right ear
1 Weber—sound is heard louder in right ear; Rinne—bone conduction exceeds air conduction in right ear Otitis media and serous otitis are potential causes of conductive hearing loss. When conductive hearing loss exists, the Weber test will result in the appearance of a louder sound in the affected ear, and the Rinne test will result in bone conduction exceeding air conduction in the affected ear. In a sensorineural hearing loss, the Weber test results in a louder sound in the unaffected ear, and the Rinne test will result in air conduction exceeding bone conduction in the affected ear.
A 7-year-old boy presents to the pediatrician's office with a 3-week history of clear nasal discharge, itchy eyes, and excessive sneezing. The family recently took in a stray cat, which sleeps with the boy. On examination of the nasal passage, you note swelling of the turbinates with clear drainage from the nares bilaterally. On examination of the oral cavity, you note slight pharyngeal erythema, post-nasal drip, and no tonsillar edema. He denies fevers, chills, nausea, and vomiting. Question - Based on the most likely diagnosis, what will appear in excess on microscopic exam of the nasal secretions? Answer Choices 1 Eosinophils 2 Neutrophils 3 Lymphocytes 4 Monocytes 5 Mast cells
1) Eosinophils - This patient presents with allergic rhinitis, supported by a history of a new pet in the house, persistent clear rhinorrhea, itchy eyes, and excessive sneezing with swollen nasal turbinates and mild pharyngeal findings. This condition typically exhibits nasal secretions rich in eosinophils, which are the specific type of leukocytes involved in allergic reactions (and parasites). Neutrophils and lymphocytes are better correlated with an infectious process. Monocytes are precursor cells that typically become macrophages, which may be present in nasal cytology but do not add much to the diagnosis. Mast cells carry antimicrobial and other inflammatory mediators and are implicated in other conditions like anaphylaxis and asthma. In non-allergic rhinitis with nasal eosinophilia syndrome (NARES), they may dominate the nasal cytology.
A 65-year-old Caucasian man presents with a 3-day history of severe dizziness. The symptoms are exacerbated by turning his head and relieved by lying still. He reports nausea and vomiting for the first 2 days of his illness but successfully eats breakfast on the day he is seen in the clinic. He denies hearing loss and tinnitus. His past medical and surgical histories are unremarkable. He has no previous exposure to ototoxic drugs and denies further neurologic symptoms. The otologic examination is without abnormality. Weber testing with a 512 Hz tuning fork is to midline. Romberg and Fukuda testing indicate right-sided pathology. Other than a crisp left-beating nystagmus, cranial nerve examination is normal. Vertigo is experienced after the Dix-Hallpike maneuver. Nystagmus is observed after a few seconds of lying down during the maneuver. Question - What is the most likely diagnosis? Answer Choices 1 Acoustic neuroma 2 Benign positional vertigo 3 Meniere's disease 4 Vestibular neuronitis 5 Viral labyrinthitis
2 Benign positional vertigo Benign positional vertigo (BPV) is the most likely cause of this patient's vertigo. BPV is not typically associated with hearing loss. Patients have brief episodes of vertigo with positional changes, typically when turning over in bed. This patient had a positive response to the Dix-Hallpike maneuver, which confirms the diagnosis. BPV is due to deposition of calcium debris in the semicircular canals. Medications such as diazepam or meclizine as well as Canalith Repositioning Procedure (Epley maneuver) are used to treat the condition. The latter is a series of head rotational positions intended to relocate free-floating particles in the semicircular canals. An acoustic neuroma (sometimes termed a neurilemmoma or schwannoma) is a benign (non-cancerous) tissue growth that arises on the eighth cranial nerve leading from the brain to the inner ear. Acoustic neuromas usually grow slowly over a period of years. They expand in size at their site of origin and, when large, can displace normal brain tissue. The brain is not invaded by the tumor, but the tumor pushes the brain as it enlarges. The slowly enlarging tumor protrudes from the internal auditory canal into an area behind the temporal bone called the cerebellopontine angle. Since the balance portion of the eighth nerve is where the tumor arises, unsteadiness and balance problems may occur during the growth of the neuroma. The most common presentation is unilateral hearing loss. The presence of episodes of vertigo along with fluctuating hearing loss is consistent with the diagnosis of Meniere's disease, which is also characterized by tinnitus or a ringing sound in the ear. Some patients also experience a pressure sensation in the ear. Episodes occur at regular intervals for years and may also be marked by periods of remission. The cause is an increase in the volume of endolymph. Vestibular neuronitis presents as acute onset of vertigo, nausea, and vomiting lasting for several days. As this condition is not clearly inflammatory in nature, neurologists often refer to it as vestibular neuropathy. The etiology is unknown, but it appears to be a sudden disruption of afferent neuronal input from the left and right inner ears. This imb
A 28-year-old woman presents with an itchy throat, prolonged sneezing episodes, red and watery eyes, and inflamed nasal membranes. Her temperature is normal and a throat culture is negative. She most likely has allergic rhinitis. Question - Which of the following factors is a common side effect of the drug that will most likely treat this condition? Answer Choices 1 Weight gain 2 Sedation 3 Constipation 4 Jaundice 5 Addiction
2 Sedation Antihistamines are the drugs that have major applications in treating the symptoms of allergic rhinitis and urticaria. They may also be used in treating motion sickness and nausea. Some antihistamines are used in the treatment of insomnia because of their strong sedative properties; therefore, they also find their way into many over-the-counter sleep aids. The most common adverse effect observed with H1 receptor blockers is sedation. Other effects seen are tremors, blurred vision, lassitude, dizziness, fatigue, drying of the nasal passages, and dry mouth. Antihistamines can interact with other drugs, potentially leading to serious consequences, such as the potentiation of the effects of CNS depressants (alcohol, etc.). Monoamine oxidase (MAO) inhibitors can potentiate the anticholinergic effects of antihistamines. In spite of this, H1 receptor blockers are relatively safe. Chronic toxicity is rare; however, acute poisoning is common, especially among children, and it leads to dangerous effects, such as hallucinations, ataxia, convulsions, and if untreated, coma and cardiorespiratory collapse. Histamine is synthesized from the amino acid histidine by a decarboxylation reaction. It is found in practically all tissues; there are large amounts in the skin, lungs, and gastrointestinal tract. It is stored in high concentrations in the granules of mast cells and basophils. It is metabolized by monoamine oxidase enzymes and by methylation. Histamine is released from mast cells in response to immunoglobulin E (IgE)-mediated (immediate) allergic reactions and plays an important role in urtication, angioneurotic edema, seasonal (allergic) rhinitis (hay fever), and anaphylaxis. It may also play a role in the control of gastric acid secretions. Histamine exerts its effect by binding to 2 major types of cell surface receptors (H1 and H2). Recently, a third type (H3) has been discovered. H1 receptors are important in producing smooth muscle effects (e.g. in bronchial smooth muscle constriction leading to decreased lung capacity and symptoms of asthma, in intestinal smooth muscle contraction resulting in cramps and diarrhea, and in exocrine glands resulting in increased nasal and bronchial mucus secretion). Bi
A 36-year-old man presents with nasal congestion, headache, fatigue, facial pain, and chronic post-nasal drip. He has had similar episodes in the past, occurring 2-3 times a year for the last several years. He has been diagnosed with acute sinusitis and antibiotics have been prescribed, providing him with relief for a brief period. This time, however, his symptoms have bothered him on and off for the last 3 months. He was given a 14-day course of antibiotics, but he experienced only partial relief. He is tired of the recurrent episodes and wants a cure. On exam, he is afebrile, nasal mucosa is inflamed, and there is mucopurulent secretion in the nasal cavity. The right maxillary sinus is tender on palpation. Lungs are clear. Question - What is the best next step in the management of this patient? Answer Choices 1) 3-week course of antibiotics 2) Plain X-ray views of the sinuses 3 CT scan of the sinuses 4 Nasal and sinus cultures 5 MRI of the sinuses
3 CT scan of the sinuses This patient has chronic sinusitis. Repeated regimens of different antibiotics have not provided him relief, and now he fits the criteria for chronic sinusitis, including 12 weeks of symptoms. Chronic sinusitis is most commonly caused by Streptococcus pneumoniae, Haemophilus influenzae, and Moraxella catarrhalis. These three bacteria account for 70% of cases. A limited CT scan of the sinuses defines the location and extent of disease and helps in deciding further management. It is quick, low cost, and sensitive. CT scanning also helps in delineating anastomotic blockage of the osteomeatal complex, so there is a role for it in cases of endoscopic surgery. A 3-week course of antibiotics may relieve symptoms briefly, but it is unlikely to cure him, especially since he already has had several courses. Amoxicillin-clavulanate or cefuroxime are traditionally used for 3 weeks. In intractable cases, a 6-week course may also be given. Clarithromycin and clindamycin are used for patients who are allergic to penicillin. Quinolones are only used if cultures show gram-negative bacteria. Plain X-rays are no longer recommended; they are not sensitive enough in the visualization of the sinuses, and they often miss findings. Nasal cultures can be contaminated with colonized organisms in the nose, such as Staphylococcus aureus, and do not correlate well with culture obtained from the sinuses. Endoscopically-guided cultures of secretions in the middle meatus or within a sinus are usually not done in clinical practice, even though occasionally it may provide the exact causative pathogen. MRI of the sinuses tells us more about the soft tissue pathology, but bony structures cannot be studied in detail. MRI is done if malignancy is suspected or if there are signs and symptoms of possible intracranial extension.
A full-term Caucasian infant is a product of an uncomplicated pregnancy, labor, and delivery. Birth weight is 8 lb, length is 21 in, head circumference is 14 in. As you examine the newborn's mouth, you identify 6 raised yellowish-white raised lesions, each approximately 1 mm in size; these lesions are located along the mid-palatine raphe. The remainder of the physical examination is normal. Question - What is the most likely diagnosis? Answer Choices 1 Alveolar cysts 2 Congenital epulis of the newborn 3 Epstein pearls 4 Fordyce granules 5 Mucoceles
3 Epstein pearls Epstein pearls are a form of cystic (fluid-filled) lesions of the palate found in up to 90% of newborns. They are located within the fusion of the posterior palatal segments. They result from the inclusion of epithelial cells during palatal fusion. The lesions are without symptoms and regress spontaneously within a few weeks to months. Alveolar cysts are small grayish-white or yellow cysts, similar in appearance to palatal cysts. They occur on the crests of the maxillary or mandibular alveolar ridges and on the facial or lingual borders of the ridges. These asymptomatic lesions regress spontaneously. Congenital epulis is a rare tumor of the newborn, occurring most commonly on the anterior alveolar ridge of the maxilla. It occurs more commonly in girls than boys (10:1). Size ranges from a few mm to several cm. It is generally a solitary firm pedunculated lesion. Fordyce granules are small slightly elevated yellow spots that may occur in clusters or may coalesce to form yellow plaques. They are typically symmetrical on each side of the mouth and occur on the buccal mucosa of the cheeks, the inner surface of the lips, and on the mucosa distal to the mandibular molars. They are not found in the newborn, generally appearing after age 10 and increasing in size and number during puberty. Mucoceles are raised fluid-filled well-circumscribed lesions several millimeters to over a centimeter in diameter usually found in the labial mucosa of the lower lip, less commonly in the upper lip, and rarely on the palate, cheek, or tongue. They occur as a result of trauma to a submucosal salivary duct, usually from blunt trauma or lip biting.
A 33-year-old man presents with a 1-day history of a painful left upper eyelid. He denies any change in vision, discharge, trauma, or foreign body. The pain started after the patient was cleaning out the garage. On physical exam, the visual acuity is OD/OS/OU = 20/20. The lateral aspect of the left upper eyelid is swollen, erythematous, and tender to palpation. The rest of the eye exam is normal. Question - What is the most likely diagnosis? Answer Choices 1 Chalazion 2 Blepharitis 3 Hordeolum 4 Dacryocystitis 5 Uveitis
3 Hordeolum - A hordeolum is an acute localized red swollen tender area on the upper or lower eyelid. A chalazion is a chronic granulomatous inflammation of a meibomian gland that is characterized by a hard non-tender swelling of the upper or lower eyelid. It is usually not erythematous. Blepharitis is a chronic bilateral inflammation condition of the eyelids. Dacryocystitis is an infection of the lacrimal sac located in the nasolacrimal area. Uveitis is an intraocular inflammation characterized by photophobia, blurred vision, and moderate pain in the eye.
A man has had 4 diagnosed sinus infections in the last 5 months, so you order a CT scan of his sinuses. He does not believe that any of the episodes completely resolved. The patient has a long history of excessive seasonal allergies; he treats them with daily oral OTC anti-histamines and he has repeatedly declined beginning allergy immunotherapy. He also has a history of asthma that is well controlled with daily inhaled corticosteroids. Question - What imaging discovery do you expect to find? Answer Choices 1 Hypodense mass 2 Mucosal thinning 3 Opacification 4 Hypoattenuation 5 Translucency
3 Opacification - This patient is most likely suffering from chronic rhinosinusitis (CRS); there may or may not be involvement of nasal polyps. The CRS would be classified as chronic due to the fact that he has had symptoms for over 12 weeks. CRS without nasal polyps accounts for up to 65% of cases of CRS. Risk factors that contribute to this condition include the presence of allergic rhinitis and asthma. On computed tomography (CT) studies, patients suffering from CRS without nasal polyps will generally present with sinus opacification (or sinus ostial obstruction) as well as mucosal thickening of the affected sinus cavity. Mucosal thinning and translucency are not associated findings on CT in CRS. Hypoattenuation generally describes an area whiter than usual and is used when referring to organs, such as the liver or kidneys.
A 4-year-old boy has had intermittent rhinorrhea, nasal congestion, and cough for about 3 weeks. His mother says he had felt warm at night when his symptoms started, and he sneezes occasionally. He goes to a large daycare 4 days a week. Otherwise, he has been healthy except for an occasional dry itchy rash that he has had on and off for "a long time." Mom has been treating this with OTC moisturizers. On exam, his temperature is 98.4°F, respirations 24, pulse 86 beats/min. He appears somewhat tired, with dark circles under his eyes. There is slightly cloudy nasal drainage, turbinates seem a little boggy, lungs are clear, ears and throat are normal and his neck is supple without any lymphadenopathy. His skin exam reveals a fine roughened slightly hyperpigmented maculopapular rash in elbow creases with a few healing excoriations. Question - What is the most likely diagnosis? Answer Choices 1 Infectious rhinitis 2 Chronic infectious rhinosinusitis 3 Vasomotor rhinitis 4 Allergic rhinitis 5 Foreign body
4 Allergic rhinitis Allergic rhinitis can frequently be mistaken for a viral or bacterial infection of the upper respiratory tract, especially in patients too young to communicate their symptoms well. Allergic rhinitis is a hypersensitivity reaction to specific allergens mediated by immunoglobulin (IgE) antibodies in sensitized patients resulting in inflammation. It can be mild or severe and may be seasonal or perennial. Allergic rhinitis also may be intermittent or persistent. Children who have one component of atopy such as asthma or eczema have a threefold greater risk of developing a second component. A variety of signs and symptoms may be present, such as mouth breathing, snoring, nasal-sounding voice, sneezing, nasal itching, snorting, nose blowing, nasal congestion and drainage, and coughing. Patients can also experience headaches, fatigue, impaired concentration, decreased sleep, and social functioning. - Classic findings include pale bluish nasal mucosa, boggy turbinates, clear nasal secretions, and pharyngeal cobblestoning. Characteristic signs include allergic shiners due to suborbital edema and allergic crease or a transverse skin line across the bridge of the nose due to constant rubbing upward. Infectious rhinitis will typically present in the early years of childhood before allergic rhinitis occurs. Nasal secretions are commonly mucopurulent; there may be a posterior pharyngeal discharge and fever. Chronic infectious rhinosinusitis also involves a mucopurulent nasal discharge, postnasal drip with cough, and an olfactory disturbance for at least 28 days. Vasomotor rhinitis consists of profuse rhinorrhea and nasal obstruction occurring with a change in environmental conditions, such as going from a warm house to frigid outdoor temperatures. Though a foreign body could be likely in this age group, especially with a history of daycare attendance, it usually will present with persistence of unilateral nasal obstruction and a purulent foul-smelling nasal discharge.
A 3-year-old child presents with sudden onset of high-grade fever, sore throat, pain during swallowing, and drooling of saliva. There is no history of a cough. The child appears toxic and is dyspneic with inspiratory stridor. The child is sitting upright and leaning forward with chin up and mouth open. Suprasternal and intercostal retractions are present. Chest is clinically clear. Blood count shows polymorphonuclear leukocytosis. Lateral radiograph of upper airway shows "thumb sign." Question - What is the most likely diagnosis? Answer Choices 1 Acute laryngotracheobronchitis 2 Bacterial tracheitis 3 Retropharyngeal abscess 4 Epiglottitis 5 Laryngomalacia
4 Epiglottitis Explanation - The most likely diagnosis is acute epiglottitis, as the child presents with sudden high fever, sore throat, dysphagia, drooling, and dyspnea. Cough is usually absent in epiglottitis. Polymorphonuclear leukocytosis and lateral radiograph of the upper airway showing thumb sign—to swelling and inflammation of the epiglottis and other supraglottic structures, especially the aryepiglottic folds—further support the diagnosis. Laryngotracheobronchitis (croup) usually starts with rhinorrhea, cough, and low-grade fever. After 2-3 days, the child develops the characteristic "barking" cough, hoarseness, and biphasic stridor. Epiglottitis has a more acute and rapid course than croup. X-ray chest shows the characteristic narrowing of the subglottic region "steeple sign." Bacterial tracheitis is more commonly caused by Staphylococcus aureus. It often follows a viral upper respiratory tract illness. Mean age is 5-7. After a few days, the child develops a brassy cough, biphasic stridor, high fever, and toxicity; the child can lie flat, does not drool, and does not have dysphagia. Respiratory obstruction may develop due to mucosal edema and thick purulent secretions, which can be adherent and membranous. Retropharyngeal abscess occurs as a complication of bacterial infection of the upper respiratory tract and commonly occurs in children ages 3-4. It occurs due to spread of infection from the upper respiratory tract to the retropharyngeal lymph nodes, which suppurate and form an abscess. It may present with fever, reduced mobility of the neck, stridor, dysphagia, and drooling. Laryngomalacia is the most common congenital anomaly of the larynx, commonly presenting with inspiratory stridor, aggravated when the infant is crying, feeding, or placed in supine position. It typically starts at age 4-6 weeks. The infant is otherwise usually happy and thriving. Stridor is due to partial collapse of a flaccid supraglottic airway.
4-year-old girl presents to the emergency department with her mother due to pain and itching in her right ear. The patient denies hearing loss and dizziness. She came back from her grandmother's house yesterday and told her mom that her ear "really itched inside." There is some drainage noted on exam from the affected ear. There is no pain when manipulating the tragus or erythema on the pinna. The mother mentions the patient and her sister were playing dress up with earrings yesterday. The mother reports the patient was inside most of the weekend and did not go swimming. Question - What is the most likely diagnosis? Answer Choices 1 Acoustic neuroma 2 Barotrauma 3 Cerumen impaction 4 Foreign body 5 Otitis externa
4 Foreign body - The clinical picture is suggestive of a foreign body because the patient complained of intense pain and itching, drainage is noted on exam, and based on the history given by mother. Acoustic neuroma presents with ringing in the ear, hearing loss, and dizziness. Barotrauma is caused by a change in pressure, which the patient did not experience. Cerumen impaction can cause ear fullness and an earache, but patients rarely exhibit intense pain or purulent drainage. Otitis externa presents with hearing loss or tragal tenderness, which is associated with swimmer's ear.
An 8-year-old male patient in third grade is referred by his school nurse to be evaluated for poor speaking and reading ability, failure to follow directions in class, and classroom disruptiveness. Despite these problems, he appears to be alert and interactive with other children and there is no demonstration of aggressive behavior or rage. At recess, he seems to play with other children as expected. He does not seem preoccupied with internal stimuli, and IQ testing results are within reference ranges. Question - What is the most likely contributing factor responsible for this patient's symptoms and behavior in school? Answer Choices 1 Autism 2 Childhood schizophrenia 3 Depression 4 Hearing impairment 5 Seizure disorder
4 Hearing impairment Hearing impairment is the most likely cause of this patient's poor communication, reading ability, and classroom problems. Children with hearing impairment appear to learn more slowly because they miss important cues and information. They often become frustrated and develop other behavioral disturbances like classroom disruptiveness. Sensory impairment is an important consideration in the differential diagnosis of any child with symptoms that might suggest intellectual disability or learning and communication disabilities. Research has shown that approximately one-third of children with hearing impairment will also be found to have at least one other disability that affects the development of speech and language (intellectual disability, cerebral palsy, craniofacial anomalies). Any child who shows developmental warning signs of a speech or language problem should have a hearing assessment by an audiologist and an examination by a geneticist as part of a comprehensive evaluation. Autism is unlikely in this case because the boy is interactive with classmates and his environment. Childhood schizophrenia is a rare condition that can cause learning disabilities and is characterized by inattention and disruptive behavior. It is unlikely in this case because there appears to be no impairment in social skills, no preoccupations with internal stimuli, and no aggression or rage behavior. Depression in children may have variable presentations. Sadness, lack of interest in activities, isolation, avoidance, change in weight may be signs. A change in school performance may occur with depression, but this child appears to be interactive with other children, and his behavior outside the classroom is not suggestive of a mood disorder. Seizure disorders (epilepsy) can cause cognitive disturbances, but these should be detectable with IQ testing. This patient has an average IQ. In addition, there are no signs or symptoms suggestive of a seizure disorder. It is a topic of much discussion and debate, but it is well known that chronic epilepsy has an association with neuropsychological impairment.
An 8-year-old child is brought to your office because of swelling of the left upper eyelid; the swelling is associated with redness and tolerable pain. No fever is noted. Physical examination shows a localized swelling and redness on the upper middle lid of the left eye; there is slight tenderness on palpation. Vital signs are within normal limits. Question - What is the most likely diagnosis? Answer Choices 1 Chalazion 2 Blepharitis 3 Entropion 4 Hordeolum 5 Ectropion
4 Hordeolum The clinical picture is suggestive of hordeolum, acute infection of the lid glands. The most common cause is Staphylococcus aureus. They are either internal (larger, extending to the conjunctival surface) or external (smaller, superficial, pointing towards the lid margins—also known as a stye). They typically self-resolve within days, responding to warm compresses, occasionally requiring topical antibiotics or drainage. They rarely extend to cellulitis, which requires systemic antibiotics. Children with frequent episodes should be evaluated for immunodeficiency. Blepharitis is an inflammation of the lid margins with redness and a scaling or crusting lesion, and symptoms of itching, irritation, burning. It is recurrent, chronic, and usually bilateral. Seborrheic type: scales are greasy, with erythema/ulceration less prominent. Staphylococcal type: ulceration is common; lashes may fall out; often accompanied by conjunctivitis or superficial keratitis. Most are mixed type. Topical antibiotics are the treatment of choice, and cleaning the surface with a moist cotton applicator is helpful. A chalazion is an inflammation of the meibomian glands characterized by a firm non-tender nodule on the upper eyelid, unlike hordeolum, which is tender/inflamed. Excision is recommended if the nodule is large enough to cause astigmatism by exerting pressure on the globe. Some cases subside spontaneously. Entropion is a condition in which the lid margin is directed inwards. It usually causes discomfort and corneal damage because the eyelashes are also turned inwards. It is most commonly caused by scarring due to inflammation seen in trachoma or Steven-Johnson syndrome. Surgery is effective. Ectropion is the opposite of entropion, in which the lid margin is turned outwards; it is associated with an overflow of tears, maceration of the lid skin, inflammation of exposed conjunctiva, and superficial exposure keratopathy. Scarring from inflammation, trauma, or muscle weakness from facial palsy are common causes. Surgical correction is necessary to protect the cornea.
A 16-year-old girl has just been diagnosed with severe allergic rhinitis caused by ragweed and dust mite. She is a candidate for allergy immunotherapy, which will involve weekly subcutaneous delivery of the offending allergens in increasing concentrations. Question - What is the ultimate goal of this type of immunotherapy for this patient? Answer Choices 1 Immunity 2 Hypersensitization 3 Immune suppression 4 Hyposensitization 5 Eradication of infection
4 Hyposensitization Allergy injections are a type of immunotherapy also known as hyposensitization. Exposure to a gradually increasing amount of allergen results in various cellular effects that lead to a decrease in the production of mast cells by the immune system. You would not want to increase hypersensitization for this patient. It does not result in complete immunity and does not suppress the immune system. Rather, it decreases the reactivity of the immune system. This treatment does not serve to treat infections, as the condition that it treats is not infectious in nature.
A 66-year-old man presents to the clinic with a complaint of not being able to hear the beeping of his microwave. Knowing that the beeping is high pitched and the age of the individual, you suspect hearing loss in this patient that is typically associated with aging. Question - This type of hearing loss is related to what alteration in the ear? Answer Choices 1 Fibrosis of the tympanic membrane 2 Hypersecretion of cerumen in the external auditory meatus 3 Ankylosis of the stapes at the oval window 4 Loss of cochlear hair cells 5 Loss of otoconia in the otolithic membrane
4 Loss of cochlear hair cells - Auditory disorders may be related to either conductive disorders or sensorineural disorders. Conductive disorders result from the mechanical impedance of sound waves from reaching the auditory sensory receptors. Sensorineural disorders result from the loss of the ability to transduce or convey the mechanical signal into the neural signal. Loss of the cochlear hair cells, particularly at the beginning of the basal turn of the cochlea, typically results in the loss of high-frequency sounds. The specific frequency is lost due to inability to transduce or convey the mechanical signal to a neural signal. This selective hearing loss of high-frequency sounds, such as that of a beeping microwave, can be associated with hearing disorders during the aging process. Loss of neurons from the spiral ganglion would be another example of a sensorineural disorder Fibrosis of the tympanic membrane, excessive secretion of cerumen in the external auditory meatus, or ankylosis (bone deposition) of the stapes at the oval window are all examples of conductive disorders leading to hearing loss. These would result in a clinical situation with the loss of sound at all frequencies rather than only a high frequency or selected frequency. The loss of otoconia in the otolithic membrane would probably have little effect on auditory responses.
A 14-year-old boy presents for a sports physical. History is significant for a high degree of myopia bilaterally first diagnosed at age 4 and a dislocated shoulder at age 10 that was easily reduced. Family history is significant for several unidentified ancestors having died in their 40s of an unidentified cardiovascular disorder. Physical examination reveals normal vital signs, height of 6'1", and weight 145 lb. The upper to lower segment ratio is 0.65 (decreased). Arm span is 76". The palate is highly arched. Mild pectus excavatum was present. A 2/6 early diastolic murmur is present best heard at the second intercostal space at the right sternal border. Arachnodactyly of the fingers and toes and generalized loose jointedness and pes planus are also present. Echocardiography reveals a tricuspid aortic valve with grade 1 (out of 4) aortic regurgitation with a normal aortic root diameter. Question - In addition to echocardiography, what evaluation would be most productive? Answer Choices 1 Brain MRI 2 Growth hormone level 3 Muscle biopsy 4 Ophthalmology evaluation 5 Rheumatoid factor
4 Ophthalmology evaluation - This patient has evidence of physical manifestations of Marfan syndrome: arachnodactyly, increased growth of long bones and abnormal long extremities, pectus deformity, high arched palate, and decreased upper-to-lower segment ratio. Aortic root disease (aortic regurgitation, dilatation of the aortic root, ascending aortic aneurysm, and aortic dissection) is the main causes of morbidity and mortality in Marfan patients. Ectopia lentis, retinal detachment, glaucoma, and other ocular anomalies are frequent in Marfan syndrome, so an ophthalmology evaluation is indicated. CNS anomalies such as intracranial aneurysms are less likely in Marfan syndrome, so screening brain MRI is not indicated. In growth hormone excess, the body habitus is proportionate, as opposed to the disproportionately long limbs seen in this patient. Growth hormone would not be elevated in Marfan syndrome. A muscle biopsy is not indicated, as the patient is not presenting with muscle weakness. Marfan syndrome is not associated with muscular abnormalities. The presentation does not suggest a collagen vascular disorder to warrant a rheumatoid factor.
A 70-year-old woman with long-standing type 2 diabetes mellitus presents due to pain in the left ear along with purulent drainage. On physical examination, the patient is afebrile. The pinna of the left ear is tender, and the external auditory canal is swollen and edematous. The white blood cell count is normal. Question - What organism is most likely to grow from the purulent drainage? Answer Choices 1 Candida albicans 2 Haemophilus influenza 3 Moraxella catarrhalis 4 Pseudomonas aeruginosa 5 Streptococcus pneumoniae
4 Pseudomonas aeruginosa - Explanation Ear pain and drainage in an elderly patient with diabetes must raise suspicion of malignant external otitis (MOE), as this infection commonly occurs in older patients with poorly controlled diabetes. The infecting organism is almost always P. aeruginosa. It can invade contiguous structures, including the facial nerve or temporal bone, and even progress to meningitis. S. pneumoniae, H. influenzae, and M. catarrhalis frequently cause otitis media, but not external otitis. Candida albicans almost never affects the external ear.
A 41-year-old woman presents to the local emergency department with a 14-day history of nasal drainage, congestion, fever, and cheek pain. She was started on amoxicillin/clavulanate initially, but she was switched to levofloxacin 7 days ago due to no improvement in symptoms. After 7 days of levofloxacin therapy, she presents again, reporting that she is no better. The fever continues and she now has upper tooth pain. On exam, she is in mild distress, with thick purulent rhinorrhea and halitosis. Question What is the next step in the care of this patient? Answer Choices 1 Admission for intravenous antibiotics 2 Continue current therapy for an additional 10 days 3 Switch to a broad spectrum antibiotic 4 Refer to ENT for sinus aspiration 5 Sinus surgery
4 Refer to ENT for sinus aspiration The patient has bacterial sinusitis that is amoxicillin/clavulanate and levofloxacin resistant. She has failed both first line and second line therapies. The sinuses need to be aspirated and cultured so that the correct antibiotic therapy can be instituted. This moderately ill patient does not warrant admission and intravenous therapy or surgical intervention. Such treatments are reserved for those who are severely ill. Since the second-line therapy of levofloxacin has had no effect after 7 days, the infection is likely resistant to this medication. Clinical trials do not support the use of broad-spectrum antibiotics for routine cases of bacterial sinusitis.
A 42-year-old patient presents with a 4-day history of worsening headache, stuffy nose, and clear-to-green nasal discharge. Therre is facial pain and a dry cough. There is no shortness of breath, abdominal pain, nausea, or vomiting. The patient does not smoke has no significant past medical history, and is only taking acetaminophen. On exam, temperature 99.2°F taken orally, pulse 86/min, blood pressure 120/76 mm Hg left arm sitting, SpO2 94% on room air. Lungs are clear and abdomen unremarkable. Nasal mucosa appears boggy, and there is tenderness with palpation over the facial bones (maxillary area). Pharynx is without exudates. Question - What agent is the most common cause of this patient's signs and symptoms? Answer Choices 1 Adenovirus 2 H. influenzae 3 Moraxella catarrhalis 4 Rhinovirus 5 S. pneumoniae
4 Rhinovirus - This scenario strongly suggests acute viral rhinosinusitis (acute sinusitis), which clinically includes green/yellow purulent discharge, facial pain or pressure over the affected sinus, nasal obstruction, congestion, cough, malaise, fever, and headache. Acute sinusitis has an acute onset of symptoms, ranging from 1-4 weeks in length of duration by the time the patient presents clinically. Signs and symptoms of bacterial and viral acute sinusitis overlap. The color of nasal discharge indicates the presence of inflammation rather than etiology. The origin of sinusitis is more often viral, and viral rhinosinusitis often originates from infection of the viral organisms that cause the common cold: rhinovirus. Other common viral organisms in adults are influenza virus and parainfluenza virus, not adenovirus. Viral signs and symptoms will resolve as time passes rather than intensifying or worsening like bacterial causes do. If suspecting a bacterial origin for cases of acute rhinosinusitis, the most common organisms that lead to this include S. pneumoniae, H. influenzae, and Moraxella catarrhalis; viral etiologies of acute rhinosinusitis are much more often seen in otherwise healthy adult patients.
22-year-old woman with a history of gradual right-sided hearing loss presents seeking help. She has been experiencing some ringing in her right ear, vertigo, and a feeling of fullness in the ear. Upon physical exam, the patient appears frustrated and keeps tugging on her right ear. MRI displays a tumor that has developed in the right inner ear. Question - What is the most likely explanation of the findings? Answer Choices 1 Acute labyrinthitis 2 Multiple sclerosis 3 Vascular compromise 4 Vestibular neuronitis 5 Vestibular schwannoma
5 Vestibular schwannoma - The clinical picture is suggestive of vestibular schwannoma, as she has symptoms of gradual right side hearing loss, vertigo, and the feeling of fullness in the ear. MRI confirmed a tumor in the right inner ear. Labyrinthitis is inflammation of the inner ear or nerves with unilateral loss of auditory function, usually associated with an ear infection that is caused by bacteria or a virus. A tumor on her MRI helps rule out labyrinthitis. Multiple sclerosis is associated with numbness, blurred vision, and weakness. Vascular compromise is blocked blood flow to an organ caused by a blocked vessel. Loss of hearing is not associated with vestibular neuronitis.
A 28-year-old man presents with diplopia and the inability to move the right eye outwards. He was hit by a ball on the right side of his face while playing volleyball 2 hours ago. His symptoms are non-progressive. On examination, his visual acuity is normal in both eyes. Right eye is medially deviated and cannot be moved laterally; otherwise, there is no abnormality detected. Question - What nerve is most likely injured? Answer Choices 1 Abducens 2 Oculomotor 3 Trochlear 4 Facial 5 Trigeminal
Abducens - The abducens, or cranial nerve VI, is the most common nerve traumatized in head injuries. It is the only supply to the lateral rectus muscle of the eye, which is responsible for external or lateral deviation of the eye. Injury to this nerve results in esotropia (internal deviation) of the eye and failure to move the eye outwards. The oculomotor, or cranial nerve III, supplies most extraocular muscles except the SO (superior oblique) and lateral rectus. Injury results in external deviation, ptosis, and pupil dilatation. The trochlear nerve supplies the SO. An injury to the nerve alone is rare, resulting in defective downward and inward gaze. The facial, or cranial nerve VII, supplies the facial muscles; an injury to the nerve results in facial paralysis (i.e., inability to close the eye, inability to frown with the forehead, and loss of mouth movement with smiling). The trigeminal nerve supplies the muscles of mastication and sensation of the face. An injury to this nerve results in defective facial sensation and mastication.
A 40-year-old man presents with severe pain in his left eye, decreased vision, and nausea. He denies recent trauma. He wears glasses for myopia. On examination, the patient's left pupil is moderately dilated and nonreactive. The cornea is "steamy" in appearance and the eye is red in general. Question - What is the most likely diagnosis? Answer Choices 1 Conjunctivitis 2 Uveitis 3 Acute angle-closure glaucoma 4 Presbyopia 5 Corneal infection
Acute angle-closure glaucoma Explanation - This patient has acute angle-closure glaucoma. The typical characteristics of this condition are all exhibited by this patient: steamy cornea, acute severe pain, blurred vision, and dilated and non-reactive pupil. Conjunctivitis does not affect pupil size; it is characterized by injection of the conjunctiva and either watery or purulent discharge. It may be viral, bacterial, or allergic. A physical exam finding in a patient with uveitis would be a unilateral small pupil that is poorly reactive. This condition can also result in decreased vision and conjunctival injection around iris (ciliary flush). Presbyopia is loss of vision due to old age; it does not affect the pupil. It is a gradual decrease in vision and not painful. A corneal infection would cause circumcorneal injection and watery or purulent discharge.
An 8-year-old girl presents with her mother to the pediatrician's office with persistent clear nasal drainage and nighttime cough for the past month. Her physical examination reveals clear rhinorrhea, dark circles under her eyes, and a transverse nasal crease. Question - What is the most likely diagnosis? Answer Choices 1 Nasal foreign body 2 Sinusitis 3 Allergic rhinitis 4 Upper respiratory infection 5 Influenza
Allergic rhinitis - Allergic rhinitis is a disorder of the nasal mucosa caused by IgE-mediated inflammation. It can present at almost any age, but incidence decreases with advanced age. In children, allergic rhinitis can occur along with atopic dermatitis and/or food allergies. The symptoms typically include persistent clear nasal drainage, stuffiness, and/or congestion. Pruritus of the eyes, nose, palate, or ears can occur. Patients often sneeze and have postnasal drainage that can lead to a bothersome cough, particularly at night. The physical examination can reveal allergic shiners, which are dark circles under the eyes. Presence of a clear nasal discharge is usually seen as well. A transverse nasal crease can be seen in children from them rubbing their noses upwards rather than using a tissue. A nasal foreign body is usually unilateral, so the drainage is also usually unilateral. The drainage would not usually be clear either, but purulent with an odor. Sinusitis would present more acutely than with a month of symptoms, and any drainage is likely to be thick and purulent. Physical examination would not likely reveal allergic shiners or a crease on the nose, but it may reveal fever, edema of the nasal mucosa, and tenderness over the sinuses. The symptoms of allergic rhinitis and upper respiratory infection (URI) may be very similar, but patients with a URI would likely not be symptomatic for a month and might have various systemic symptoms. A nasal crease and allergic shiners are also not likely to be present. Patients with influenza would not have symptoms for a month and would usually have systemic symptoms, such as malaise, fever, and arthralgias. The cough would usually be more significant than a bothersome nighttime cough secondary to postnasal drip. A nasal crease and allergic shiners are also not likely to be present.
A 10-year-old boy presents due to a lesion on his gums. For the past 2 days, he has reported soreness around his teeth and inside his cheeks. When his mother looked closely in his mouth with a flashlight, she noted bright red areas and patches of white curd-like lesions. There are no ill contacts in the last few weeks. There has been no fever or shortness of breath. The patient's past medical history is positive for severe seasonal allergies and mild persistent asthma. Daily medications include: pediatric multivitamin, cetirizine hydrochloride 10 mg 1 tablet at bedtime, montelukast sodium 5 mg 1 chewable tablet in the morning, beclomethasone disproportionate HFA 40 mcg 2 puffs twice daily in the am and pm, and albuterol sulfate inhalation powder 2 inhalations every 4-6 hours as needed for wheezing and 15 minutes before physical activity. Examination reveals adherent thick white plaques with underlying erythematous tender mucosa on the gingival and buccal surfaces. Question - What medication would be most appropriate to prescribe? Answer Choices 1 Amoxicillin suspension 2 Azithromycin suspension 3 Chlorhexidine gluconate 4 Fluticasone propionate 5 Nystatin suspension
Answer: Nystatin suspension - These lesions are highly likely to be caused by Candida albicans, which commonly causes oral candidiasis (thrush). The lesions are adherent creamy white plaques on the buccal, gingival, or lingual mucosa; they may be asymptomatic or painful. Thrush is a common condition in infants, especially during the first few weeks of life. Other reasons pediatric patients will develop this condition includes recent antimicrobial therapy and daily inhaled corticosteroid therapy, such as for asthma maintenance. It is highly likely the cause of this patient's thrush. Educating patient and his mother about washing/swishing his mouth immediately after using beclomethasone with water or mouthwash will easily eliminate this risk. Treatment in uncomplicated cases is typically nystatin suspension 200,000-500,000 units to be swished like mouthwash and spat out 5 times daily. Other treatment options for pediatric patients include gentian violet, clotrimazole troches, and fluconazole. Azithromycin and amoxicillin are antibiotics for bacteria where this patient's diagnosis is fungal in origin. Chlorhexidine gluconate is most commonly used as a treatment for gingivitis, which would not treat this patient's symptoms. Fluticasone propionate is a corticosteroid, which is what most likely caused the issue to begin with.
A 22-year-old woman presents with a 1-year history of flaking and scaling around her lashes, along with itching and a burning sensation. She has also noted her lid margins are red, and some of her lashes are missing. Her history is significant for diabetes and for seborrheic dermatitis of the scalp, eyebrows, and external ears. What is the most likely diagnosis? - Blepharitis - Chalazion - Conjunctivitis - Ectropion - Foreign body
Blepharitis Blepharitis may be seborrheic or ulcerative. Seborrheic (non-ulcerative) blepharitis is commonly associated with seborrhea of the face, eyebrows, external ears, and scalp. Inflammation of the eyelid margins occurs, with redness, thickening, and often the formation of scales and crusts or shallow marginal ulcers. Ulcerative blepharitis is caused by bacterial infection (usually staphylococcal) of the lash follicles and the meibomian glands. Removal of crusts, topical antibiotics, and/or oral antibiotics remains the mainstay of treatment. Chalazion is a swelling of the eyelid margin due to granulomatous inflammation; it usually resolves spontaneously with warm soaks and time. Chalazions are usually painless. Multiple chalazia are seen in diabetes, alcoholism, and malnourishment. In extreme cases, curettage may be required. Conjunctivitis can be caused by bacterial, viral, allergic, and irritant etiologies. Clients usually complain of red eyes and a sticky or watery discharge. The type of discharge helps to determine the etiology; it is watery in viral, and sticky green or yellow in bacterial conjunctivitis. Irritation is common, but severe pain and photophobia are not. Bacterial or viral conjunctivitis is usually self-limited, but it may be treated with a topical antibiotic, without steroids, such as sulfacetamide (10% 3-4 times/day). Topical aminoglycoside should be reserved for more refractory disease. Allergic conjunctivitis may be effectively treated with a new class of non-steroidal, topical, anti-inflammatory agents. Irritant conjunctivitis, including dry eyes, may be treated with topical, non-preserved lubricants. Ectropion is the outward turning of, usually, the lower lid occurring in older people. Surgery is indicated if ectropion causes excessive tearing, exposure, keratitis, or a cosmetic problem. Foreign body sensation is most commonly due to corneal or conjunctival foreign bodies. Other causes are disturbances of the corneal epithelium and rubbing of eyelashes against the cornea (trichiasis).
A 52-year-old man presents with a burning sensation in both eyes. He denies recent trauma and contact with individuals who have similar symptoms. On examination, his eyelid margins are red and inflamed. His eyelashes are greasy and adherent with a surrounding dandruff-like scale. Conjunctivae are clear. Question - What is the most likely diagnosis? Answer Choices 1 Hordeolum 2 Chalazion 3 Conjunctivitis 4 Blepharitis 5 Pinguecula
Blepharitis - Blepharitis is a chronic inflammation of the lid margins. Causes include infection, seborrhea, and meibomian gland dysfunction. Blepharitis characteristically causes red eyelid margins with adherent eyelashes and scale-like deposits. It is treated with eyelid scrubs with diluted baby shampoo. Antibiotics may be used if infection is suspected. Hordeolum presents as a small inflammatory nodule in a gland on the upper or lower eyelid margin. Chalazion presents as a painless lesion of the palpebral margin. Conjunctivitis presents in several different ways depending on the etiology, but the conjunctivae would be erythematous; this patient's conjunctivae are clear. Pinguecula presents as an elevated yellow fleshy mass on the sclera adjacent to the conjunctiva.
A 23-year-old woman presents to the emergency department after a baseball hit her in the right eye. She has double vision, pain upon moving the right eye, and numbness of the cheek, nose, and right side of the eye. Upon physical exam, swelling, tenderness, numbness of the nose, and epistaxis are present. Question -What is the most likely diagnosis? Answer Choices 1 Blepharitis 2 Blowout fracture 3 Conjunctivitis 4 Corneal abrasion 5 Stye
Blowout fracture - The patient's history and presentation are suggestive of blowout fracture (orbital floor fracture). X-ray would confirm a fracture. Blepharitis is when oil glands become clogged or irritated. A patient would present with itchy eyelids, watery eyes, a feeling that something is in their eye, or crust on the eyelashes. Conjunctivitis presents with burning, itching, irritation, and discharge. Corneal abrasion does not present with swelling, double vision, and numbness of the cheek. It is very painful due to the scratch on the surface of the cornea. It is diagnosed with fluorescein stain. Styes occur when bacteria gets into the oil gland in the eyelids, which creates a red bump located closer to the eyelashes. It can make the eye feel watery or produce a feeling that something is in the eye.
A 35-year-old woman who recently finished a round of oral antibiotics for pneumonia presents due to her tongue having "a thick white coating"; she adds that it is "also red and irritated." She is able to "scrape the white stuff off" some areas of her tongue, which she reports have become raw and more erythematous. Question - What is the most likely diagnosis? Answer Choices 1 Hairy leukoplakia 2 Atrophic glossitis 3 Hairy tongue 4 Geographic tongue 5 Candidiasis
Candidiasis - Candidiasis is an infection that may cause the tongue to have a white coating. This coating can be scraped and a sample can be analyzed for the presence of Candida. Hairy leukoplakia may be seen in people infected with HIV and AIDS. It is characterized by raised areas that are whitish-tan in color and have a feathery appearance. Hairy leukoplakia cannot be scraped off. A "hairy tongue" is not actually due to hair growth on the tongue; it consists of elongated papillae that have the appearance of grayish-black hair to the naked eye. This condition may be caused by antibiotic use, or there may not be any reason. Atrophic glossitis (or smooth tongue) presents as having a smooth surface due to papillae loss. The loss may indicate deficiency in riboflavin, niacin, folic acid, vitamin B12, pyridoxine, or iron. A geographic tongue is a benign condition with unknown cause; it is characterized by a map-like pattern of smooth, red areas that do not have papillae as well as rough areas that still have papillae.
A 78-year-old Caucasian man presents with unilateral painless loss of vision in the right eye of 3 hours duration. Examination reveals an elderly man who is anxious but in no acute distress. Visual acuity is light perception only in the right eye and 20/30 in the left eye. Pupillary examination is significant for an afferent pupillary defect on the right side. Penlight examination of the eyes is otherwise unremarkable. Retinal examination of the right eye reveals a cherry-red spot. Retinal examination of the left eye is unremarkable. Question What disease process most likely accounts for the patient's presentation? - Adult-onset Tay-Sachs Disease - Open-angle glaucoma - Central retinal Artery occlusion - Trauma - Cataract
Central Retinal Artery Occlusion - This case represents the classic presentation of a central retinal artery occlusion. It is acute, unilateral, and painless; there is loss of vision as well as a cherry-red spot on fundus examination. Tay-Sachs disease is a lysosomal storage disease found predominantly in the Ashkenazi Jewish population. Infants with this fatal neurodegenerative disease have a cherry-red spot on retinal examination. An adult-onset form of Tay-Sachs disease is rare but does exist. Onset of symptoms is in the 20s and 30s, but it is characterized by neurologic deterioration and cherry-red spots would be bilateral. Although this patient does have a cherry-red spot, there is nothing in the presentation to suggest Tay-Sachs disease. Open-angle glaucoma is a chronic, slowly progressive condition that would not be expected to cause acute visual loss. The patient gives no history of trauma, and no evidence of trauma is seen on examination. A cataract, or opacification of the lens, would not be expected to cause acute visual loss.
A 64-year-old African American man presents to the emergency department after he went blind in his right eye "out of the blue" 20 minutes ago. There is no pain associated with his symptoms and he is not nauseated. Past medical history is positive for type 2 diabetes mellitus for the past 10 years. The pupil reaction on the left side is normal with pressure of 17 mm Hg. Right pupil evaluation reveals no reaction to light or accommodation with pressure of 20 mm Hg. Right eye ophthalmoscopy reveals arteriolar narrowing, vascular stasis, and "boxcar" pattern. Question - What is the most likely diagnosis? Answer Choices 1 Central retinal artery occlusion 2 Acute glaucoma attack 3 Subconjunctival hemorrhage 4 Retinal detachment 5 Macular degeneration
Central retinal artery occlusion - The symptoms described are typical for occlusion of the central retinal artery, which is a branch of the ophthalmic artery, in turn a branch of the internal carotid artery. The "boxcar" pattern is segmentation of the venous blood column; bilateral boxcar ring is a useful sign of circulatory arrest and death. Acute central artery occlusion is an emergency, as it results in permanent blindness if circulation is not restored within 30-60 minutes. - An acute glaucoma attack is accompanied by severe pain with decreased vision. The patient usually reports seeing halos around light. The pupil is fixed in a mid-dilated position, and the eyeball is firm to pressure since the intraocular pressure is elevated. - Subconjunctival hemorrhage onsets spontaneously and shows a painless bright red patch on the sclera. It usually is caused by overexertion; it is benign and self-limited with no influence on vision. - Retinal detachment starts with the patient seeing dark vitreous floaters, light flashes, and blurred vision, which progresses to blindness if not treated. -Macular degeneration causes painless loss of visual acuity. There is altered pigmentation in the macula.
A 42-year-old man presents with a firm painless bump on his left upper eyelid. On examination, you note a 5 mm mass within the tarsus of the left eye. The skin is freely movable over the mass. The remainder of the eye exam is unremarkable. Question: What is the most likely diagnosis? Answer Choices 1 Pterygium 2 Ectropion 3 Hordeolum 4 Chalazion 5 Xanthelasma
Chalazion The clinical picture is an example of a chalazion. A chalazion is a usually painless chronic mass in the eyelid. Ectropion is when the eyelid sags outwardly and the lid does not close well. Hordeolum is acute, red, and painful. - "It hurts like a hore" Pterygium involves the sclera. Xanthelasma is a yellow plaque filled with cholesterol that usually appears on the eyelids.
An 18-year-old girl has had a small slightly tender swelling in her left upper eyelid for 10 days. For the first day or two, it was red and a little painful. Now it is painless, but it has grown in size. There has not been any drainage, visual changes, or itching noted. She has been well otherwise. On exam, vitals are normal, extraocular muscles are intact, and pupils are equal and reactive to light. Her left upper eyelid has a 1.5 cm round non-tender swollen mass that is mildly erythematous without any drainage. The underside of the lid is grayish-red. Question - What is the most likely diagnosis? Answer Choices 1 Chalazion 2 Blepharitis 3 Dacryocystitis 4 Dacryostenosis 5 Hordeolum
Chalazion A chalazion is an enlargement of a deep oil gland in the eyelid that results from an obstruction of the gland opening at the edge of the eyelid. Most will disappear without treatment after 1-3 months. If hot compresses are applied several times a day, they may disappear sooner. Persistent lesions that cause changes in vision should be examined by an ophthalmologist for drainage by incision and curettage or intralesional corticosteroid injection. Hordeolum (stye) is an acute infection of one or more of the glands along the edge of the eyelid or under it, usually caused by a staphylococcal infection. A stye will usually last 2-4 days. It begins with redness, tenderness, and pain at the edge of the eyelid. A small round tender swollen area often forms with a tiny yellowish central spot. Blepharitis is an inflammation along the edges of the eyelids with possible thickening scales, crusts, shallow ulcers, or inflamed oil glands. The eyes and lids may itch and burn, or they may become watery and photosensitive. The eyelid may swell and become red. Dacryostenosis results in a blockage of the flow of tears from the eye to the nose due to narrowing of the nasolacrimal ducts. This can result from inadequate development of any part of the ducts, chronic nasal infection, severe or recurring eye infections, or fractures of the nasal or facial bones. Dacryocystitis is an infection of the lacrimal sac that results from a blockage of the nasolacrimal duct, which leads from the lacrimal sac into the nose. The area around the lacrimal sac becomes painful, red, and swollen. Pressure to the lacrimal sac may push pus through the opening near the nose. Fever is common.
A 10-day-old male newborn presents with bilateral conjunctivitis with moderate white discharge. He is acting normally, has no fever, and is feeding well. He was born full-term without any complications. His mother had minimal prenatal care. He has been gaining weight well. On exam, he is alert and active. Culture with immunofluorescence reveals inclusion bodies. Question - What is the most likely cause of this newborn's conjunctivitis? Answer Choices 1 Chlamydia trachomatis 2 Herpes simplex 3 Neisseria gonorrhoeae 4 Silver nitrate 5 Coxsackievirus
Chlamydia trachomatis - Chlamydia and gonorrhea are the most common causes of conjunctivitis neonates. Given the timing and presentation, chlamydia is the most likely cause of this conjunctivitis. A characteristic finding of chlamydial infection is the presence of inclusion bodies in the epithelial cells of a conjunctival smear. The usual incubation period for C. trachomatis is 5-14 days and 2-5 days for N. gonorrhoeae. Gonococcal conjunctivitis tends to produce a more purulent discharge than C. trachomatis. Herpes conjunctivitis is sometimes contracted when neonates are born to mothers with herpes lesions in the genital region. Conjunctivitis due to silver nitrate drops usually occurs 6-12 hours after birth. Coxsackievirus is not a common cause of conjunctivitis in neonates.
A 3-day-old female newborn presents at your outpatient clinic with rapidly progressing bilateral conjunctivitis with white discharge. She was born full-term via precipitous vaginal delivery; her mother had no prenatal care. On exam, she is alert and active. The eye discharge is purulent and she has bilateral eyelid edema. Question - What is the most crucial next step in the management of this newborn? Answer Choices 1 Pursue past medical history at greater length with mother prior to treatment. 2 Give an IM dose of ceftriaxone and re-evaluate in 24 hours. 3 Start topical erythromycin with frequent eye flushing. 4 Start nasolacrimal duct massage and follow clinically. 5 Admit to the hospital for IV antibiotics and evaluation
Correct Answer: Admit to the hospital for IV antibiotics and evaluation. - Ophthalmia neonatorum is a form of conjunctivitis occurring in newborns (under 4 weeks). The usual incubation period is 2-5 days for N. gonorrhoeae, and 5-14 days for C. trachomatis. Conjunctivitis due to silver nitrate drops usually occurs 6-12 hours after birth. The condition may also be caused by a virus, most commonly herpes simplex. Due to the timing and severity of conjunctivitis, in conjunction with the fact that the mother had no prenatal care, it is possible that this newborn has gonococcal conjunctivitis. Due to the possibility of blindness, gonococcal meningitis, or sepsis, the patient should be monitored closely and admitted to the hospital for eye, blood, and (possibly) CSF cultures. Tests for other infections (e.g., C. trachomatis, syphilis, hepatitis B, and HIV) should be performed. Pursuing past medical history and mother prior is moot at this point, as the diagnosis must be dealt with in a timely manner. Giving an IM dose of ceftriaxone is not enough, and it would be inappropriate to send this patient home as an outpatient at this point in care due to the high potential of permanent complications. Topical treatment with erythromycin and eye flushing would be ineffective and not considered appropriate plan of care. Nasolacrimal duct massage is more for an anatomical variation, not for an acute infectious process.
A 28-year-old man presents with "burning mouth." The patient states he has had an unusual burning sensation on the inside of his right cheek for the last 3 days; he feels that the burning is worsened with hot, spicy, or acidic foods. After performing a comprehensive oral exam, you note three lesions on the buccal mucosa on the right side of the oral cavity. You further document the description of these lesions as the following: three round lesions, each measuring approximately 3 mm in diameter with presence of a white-yellow center surrounded by a red halo. Question - What is the most likely diagnosis? Answer Choices 1 Aphthous ulcers 2 Fordyce spots 3 Koplik's spots 4 Leukoplakia 5 Oral candidiasis
Correct Answer: Aphthous ulcers - This patient is experiencing an episode of aphthous ulcers (canker sores, aphthous stomatitis). These painful open sores are found in the oral cavity and are the most common form of mouth ulcer. They are benign, non-cancerous, and non-infectious, and in most cases the cause is unknown. The lesions are often described as having a white or yellow center surrounded by a bright red area. Fordyce spots (Fordyce granules) are sebaceous glands that appear as small yellowish spots in the buccal mucosa or on the lips. Fordyce spots are a benign finding. Koplik's spots are a sign indicating measles: small white or bluish specks that resemble grains of salt on a red background. They often appear on the buccal mucosa near the first and second molars. Leukoplakia is described as a thickened white patch anywhere in the oral mucosa that results from chronic local irritants, such as chewing tobacco. This istypically a benign reactive process, but it can eventually lead to a malignant process and should be biopsied. Oral candidiasis (thrush) lesions usually consist of creamy-white patches overlying erythematous mucosa that are notably painful to the patient. The white patches usually can be rubbed off. Although thrush may be encountered in infants and babies, the following adult patient populations commonly encounter this pathology: those who wear dentures, those with prolonged history of poor oral hygiene, with diabetes, with anemia, those using antibiotic therapy, those undergoing chemotherapy or local radiation, and those using daily corticosteroids.
A 10-year-old boy presents due to a lesion on his gums. For the past 2 days, he has reported soreness around his teeth and inside his cheeks. When his mother looked closely in his mouth with a flashlight, she noted bright red areas and patches of white curd-like lesions. There are no ill contacts in the last few weeks. There has been no fever or shortness of breath. The patient's past medical history is positive for severe seasonal allergies and mild persistent asthma. Daily medications include: pediatric multivitamin, cetirizine hydrochloride 10 mg 1 tablet at bedtime, montelukast sodium 5 mg 1 chewable tablet in the morning, beclomethasone disproportionate HFA 40 mcg 2 puffs twice daily in the am and pm, and albuterol sulfate inhalation powder 2 inhalations every 4-6 hours as needed for wheezing and 15 minutes before physical activity. Examination reveals adherent thick white plaques with underlying erythematous tender mucosa on the gingival and buccal surfaces. Question - What medication has increased this patient's risk for developing this condition? Answer Choices 1 Albuterol sulfate 2 Beclomethasone dipropionate 3 Cetirizine hydrochloride 4 Montelukast sodium 5 Multivitamin
Correct Answer: Beclomethasone dipropionate - These lesions are highly likely to be caused by Candida albicans, which commonly causes oral candidiasis (thrush). The lesions are adherent creamy white plaques on the buccal, gingival, or lingual mucosa; they may be asymptomatic or painful. Thrush is a common condition in newborns, especially during the first few weeks of life. Pediatric patients may also develop this condition due to recent antimicrobial therapy and use of daily inhaled corticosteroid therapy, such as for asthma maintenance (beclomethasone dipropionate). Educating patient and his mother about washing/swishing his mouth immediately after using beclomethasone with water or mouthwash will easily eliminate this risk. Less commonly, but still possible, is the patient potentially having a positive HIV status; this is not a high possibility in this patient. Albuterol sulfate and montelukast sodium help control asthma symptoms. Albuterol inhalers may cause throat irritation but is not commonly associated with thrush. Cetirizine hydrochloride is used to relieve seasonal allergies symptoms and is not associated with developing thrush nor would a daily multivitamin increase this patient's risk of developing thrush.
A 78-year-old woman presents with behavioral changes over the past few months. Her daughter feels that her mother is depressed, as she talks less frequently with her family members. She was a regular at various social gatherings, but the patient has been avoiding them lately due to some difficulty in communication. She has been spending more time painting portraits and has been watching television at a higher volume than usual. She has no suicidal thoughts. She is well oriented to time, place, and person; her mood today seems good. Ear examination and tests indicate sensorineural hearing loss. The rest of her physical examination is within normal limits. Question - What is the best initial step in treating her condition? Answer Choices 1 Supportive counseling 2 Cochlear implants 3 Antidepressant medications 4 Bilateral hearing aids 5 Psychotherapy
Correct Answer: Bilateral hearing aids - This patient has presbycusis. She avoids social gatherings, as patients with her condition have difficulty hearing, particularly in noisy environments. It does affect self-esteem, so it can be confused with depression. She has not lost interest in other activities (like painting portraits), however, which helps rule out depression. Presbycusis is a multifactorial sensorineural loss initially affecting high frequencies and becoming progressively worse over decades. Presbycusis is the most common form of hearing loss affecting older adults (over 50). Ear examination, including pinnae, canals, and tympanic membranes, appears normal. Audiometry reveals bilateral symmetric moderate-to-severe sensorineural hearing loss, more pronounced in the higher frequencies. Hearing aids are used to amplify sounds, though they cannot fully restore the quality of hearing. Cochlear implants are a choice when hearing aids prove inadequate. Psychotherapy, supportive counseling, and antidepressant medications are used for patients diagnosed with depression.
A 74-year-old man presents with a 90-minute history of severe pain and blurred vision in his left eye. He reports headache and some nausea along with halos surrounding lights. Upon examination, his left eye is erythematous with a steamy cornea and a poorly reactive mid-dilated pupil. An ophthalmologic consult is ordered and tonometry is completed, revealing an elevated intraocular pressure. Gonioscopy was used to confirm the diagnosis and examine the fellow eye, showing narrow anterior chambers in both eyes. Question - What will be the definitive treatment for this patient after acute attack resolution? Answer Choices 1 Left laser peripheral iridotomy 2 Bilateral laser peripheral iridotomy 3 Intravenous acetazolamide 4 Oral hyperosmotic glycerin 5 Topical timolol 0.25%
Correct Answer: Bilateral laser peripheral iridotomy - Bilateral laser peripheral iridotomy is a procedure involving a puncture-like opening made near the base of the iris to decrease intraocular pressure in patients with angle-closure glaucoma. While there are various medications used to treat acute episodes, this procedure will correct the disorder definitively where the medications are temporary treatment. Patients with narrow anterior chambers are at risk for angle-closure glaucoma. Both eyes are affected in this case, so bilateral iridotomy is the best option. Left laser peripheral iridotomy will correct the disorder definitively, but patients with narrow anterior chambers are at risk for angle-closure glaucoma. Narrow anterior chambers occur in the fellow eye 50% of the time; this patient's ophthalmologic exam confirms narrow anterior chambers in both eyes, so treatment should be considered for both eyes. Intravenous acetazolamide is given in episodes of acute angle-closure glaucoma to decrease intraocular pressure. It is typically given in a single 500 mg IV dose. This is effective to control the acute episode, but it will not treat the disorder definitively; this patient's underlying issue is narrow anterior chambers. Oral glycerin is a hyperosmotic agent that can be given 1-2 g/kg/dose to decrease a patient's intraocular pressure during an acute episode of angle-closure glaucoma in cases with severe increased IOP and when medical therapy is unsuccessful. This is effective to control the acute episode, but it will not treat the disorder definitively; the patient's underlying issue is narrow anterior chambers. Topical timolol 0.25% is a topical β-adrenergic blocking agent used 1-2 times per day to lower IOP in combination with pilocarpine and timolol in acute angle closure glaucoma. This medication is used in an acute attack, but it is not a definitive treatment.
73-year-old man presents with a nosebleed that will not stop. The bleeding has been present for over 2 hours. The patient's nose began dripping blood at breakfast; there was no known trauma. He denies pain. He has tried applying nasal pressure and lying down to rest. Until the bleeding began, the patient had not been experiencing any nasal symptoms, such as congestion, impaired nasal patency, or rhinitis. Because he felt too dizzy to drive, his wife drove him to the emergency department. The patient has no diagnosed medical conditions and takes no medications. He denies prior episodes of severe nosebleeds, easy bruising, and any known bleeding disorder. The patient's vitals are shown in the table. weight - 148lb, Height - 69', P-120, RR-18, BP- 90/66, T- 97.4F/36.3c On physical exam, the man is holding a large bloodsoaked towel to his nose, with continued brisk bleeding. He is otherwise in no apparent distress but seems somewhat confused. On rhinoscopy, bleeding is observed from bilateral nares. No foreign body, mass, lesions or abrasions are visualized. The bleeding site cannot be identified. Question - What test should be ordered next for this patient? Answer Choices 1 Blood type
Correct Answer: Blood type and Rh - This patient most likely has a posterior epistaxis. Posterior bleeds are less common than anterior bleeds, but they should be suspected when bleeding is high in volume and unresponsive to nasal packing/tamponade. The cause of this patient's posterior bleed is not evident; many are idiopathic. With confusion, tachycardia, tachypnea, hypotension, and blood loss, however, the provider must recognize that this patient is presenting with (hypovolemic) shock, which must be addressed before the epistaxis. If this patient needs a transfusion, a blood type and Rh would be appropriate. If a patient presents with shock and obvious blood loss, first-line actions include control of bleeding, crossmatching of blood, and infusion of fluids and blood products. Coagulation studies are rarely helpful in evaluation of epistaxis unless the episodes are recurrent or there are other reasons to suspect a coagulopathy. Because the patient is 73, it would be unusual for him to present with a coagulation disorder for the first time in his life. Computed tomography (CT) of the sinuses or plain nasal X-ray could be useful in evaluating patients with epistaxis and a history of trauma. Imaging may show a fracture or a tumor (in rare cases). This patient denies any trauma, however, and he needs vascular stabilization before imaging. Nasal endoscopy is accomplished using a flexible or rigid fiber-optic endoscope to view the nasal and sinus passages. It can be useful in locating the origin of the bleed in epistaxis. Once this patient is vascularly stable, endoscopy may be useful.
An 8-month-old boy is admitted due to possible middle ear infection and chronic diarrhea. He has episodes of frequent loose stools since birth. He had two episodes of lower respiratory tract infection with bronchospasm, treated with intravenous antibiotics and bronchodilators. Parents are not related, and their firstborn died of pneumonia at age 8 months. The patient appears irritable, screams, shakes his head, and tries to rub the right ear. Signs of dehydration are noted. Weight is <5th percentile, height is 5th percentile, and head circumference is 50th percentile. Temperature is 38°C. Mucocutaneous changes suggestive of a fungal infection are noted. Right tympanic membrane is erythematous and bulging, with poor mobility on pneumatic otoscopy. Left tympanic membrane appears clear with good mobility. Throat is erythematous. Thyroid, heart, lungs, genitalia, and nervous system are within normal limits; abdomen is tender. You cannot palpate lymph nodes, and there are no signs of hepatosplenomegaly. Question - What is the next step in the diagnosis of the primary disorder in this child? Answer Choices 1 KOH prep of skin changes 2 CBC with differential 3 Genetic testing 4 Testing stoo
Correct Answer: CBC with differential - This patient most likely has severe combined immunodeficiency (SCID), a group of congenital diseases caused by different genetic mutations, resulting in the severe deficiency of both T and B lymphocytes. X-linked is the most common type. The clinical picture is similar among them: recurrent infections caused by bacteria, viruses, fungi, and opportunistic infections. The initial step in the evaluation of immune status should be a complete blood count with the differential. You should evaluate blood cell counts and cell morphology for the presence of lymphopenia, the classic hallmark of SCID. Lymphocyte markers should be obtained to check percentages and absolute counts of T cells, B cells, and natural killer cells. - KOH prep may help differentiate dermatophytes and Candida albicans symptoms from other skin disorders, but it will not be useful in the diagnosis of the primary condition. -You may consider genetic testing to differentiate between various forms of SCID and the other combined immune deficiencies, but testing should only be done after you prove the existence of immune deficiency (lymphocytes, immune globulins). - Testing stool for ova and parasites may be considered in cases of diarrhea, but the testing will not help you to establish the diagnosis of this patient's primary condition. - The patient probably has otitis media, and he may have mastoiditis (peak incidence at age 6-13 months), which is much more common in immunocompromised patients. In the case of mastoiditis, fluid extracted from the middle ear through either perforated drums or by tympanocentesis may be sent for Gram staining, culture, and acid-fast stain, but this procedure will not contribute to the evaluation of immune deficiency.
A 24-year-old man presents with a painless, localized swelling of his left lower eyelid; it has developed over a period of weeks. He is seeking medical attention because it is now producing a foreign body sensation in his left eye; it is also hindering his path of vision. On physical examination, his visual acuity is normal; there is no evidence of injection or discharge. You palpate, and you observe a nontender, localized nodule on the lower eyelid. Question - What is the most likely diagnosis? Answer Choices 1 Keratitis 2 Chalazion 3 Stye 4 Blepharitis 5 Ectropion
Correct Answer: Chalazion - A chalazion is a granulomatous inflammation of the meibomian gland. It typically presents as a hard non-tender swelling of either the upper or lower lid, sometimes accompanied by redness and swelling of the adjacent conjunctiva. Typically, blepharitis is a bilateral condition of general inflammation of the eyelid skin, eyelashes, and associated glands. Characteristics of blepharitis include red-rimmed eyes and scales on the eyelashes. Tears may even have a greasy distinction to them. Ectropion is the outward turning of the lower lid. Keratitis produces a painful eye, a hazy-appearing cornea as well as evidence of an ulcer or even an abscess in this area; hypopyon is also possible in patients who have keratitis. Stye or hordeolum is a painful swelling of the eyelid associated with inflammation and erythema.
A 37-year-old man presents with a feeling of constant left ear fullness. His hearing has not been as sharp out of his left ear for the past 3 months. The patient had multiple middle ear infections as a child and had myringotomy tube placements on four separate occasions. He has a history of severe perennial and seasonal airborne allergies, but he is not seeking treatment currently. Weber test findings lateralize to the left ear. During otoscopic inspection, a sac filled with a cheesy white material is located on a retracted tympanic membrane. A small perforation of the tympanic membrane is suspected; there is the presence of the same cheesy white debris behind the membrane. Question - What is the most likely diagnosis? Answer Choices 1 Barotrauma 2 Cholesteatoma 3 Exostoses 4 External otitis 5 Gradenigo syndrome
Correct Answer: Cholesteatoma - Several components of his history increase the chances of development of cholesteatomas. Chronic, severe airborne allergens have most likely led to chronic eustachian tube dysfunction, the most likely cause of cholesteatomas. The patient has had numerous middle ear infections—a very specific variation of otitis media. Cholesteatomas are squamous epithelium-lined sacs usually filled with desquamated keratin, which eventually becomes chronically infected. The abnormal findings of the Weber test (left lateralization) indicate that this patient most likely has some invasion of the contents into at least the middle ear and ossicles, leading to conductive hearing loss. These sacs can cause erosion of surrounding structures, including the tympanic membrane (rupture), mastoid, and ossicles, eventually disrupting the function of the inner ear and the facial nerve. Treatment is typically surgical marsupialization of the sac. Barotrauma is a pathology that also results from eustachian tube dysfunction, but it is mainly caused by extreme barometric changes in pressure (eg, air travel, underwater diving, rapid altitudinal change). Exostoses (typically benign bony overgrowths of the ear canal) and otitis externa (swimmer's ear, outer ear infection) are not evident in this patient. Gradenigo syndrome is characterized by the triad of retro-orbital pain, acute otitis media, and ipsilateral abducens nerve paralysis—an extremely rare complication.
An 8-year-old boy presents with left ear pain. His father reports that he had two ear infections as a baby, but he cannot remember which ear. The visit occurs during the summer months, and the father says that the patient has been swimming almost daily in a neighbor's pool. Physical examination of the ears bilaterally reveals left ear canal erythema and edema and pain with manipulation of the left pinna. No other physical examination findings are abnormal. Question - What is the most likely treatment? Answer Choices 1 Ciprofloxacin/dexamethasone topical solution 2 Clotrimazole 1% topical solution 3 Ciprofloxacin oral suspension 4 Triamcinolone 0.1% topical solution 5 Amoxicillin oral suspension
Correct Answer: Ciprofloxacin/dexamethasone topical solution - This patient has acute left bacterial otitis externa. Initially, any desquamated epithelium and moist cerumen should be removed. Topical solutions are usually adequate to treat the infection and should be chosen based on the most likely causative organism. Bacterial cases of otitis externa are usually caused by Pseudomonas aeruginosa or Staphylococcal species. Topical therapy containing a steroid is beneficial to decrease inflammation. First-line agents include ciprofloxacin/dexamethasone solution or neomycin/polymyxin B/hydrocortisone topical solution, but some studies show superiority of quinolones compared to non-quinolones. It is usually given 3-4 times daily for 7-10 days. Clotrimazole 1% would be adequate for treatment of fungal otitis externa, as clotrimazole is an antifungal rather than an antibacterial medication. Topical treatments are usually enough to resolve episodes of acute otitis externa. Should it become recurrent or if signs of invasive infection (such as fever or cellulitis) are present, oral antibiotics may be necessary. In those cases, fluoroquinolones are often chosen due to coverage of Pseudomonas species. Due to musculoskeletal related toxicity, routine use of fluoroquinolones should be avoided in children; as recommended by the American Academy of Pediatrics, a systemic fluoroquinolone can be used in children when no alternatives exist. Amoxicillin would not provide coverage against Pseudomonas. Some children have eczematous otitis externa, and this should be treated differently than acute bacterial otitis externa. Topical therapy is still effective, but acetic acid 2%, aluminum acetate, or various steroid preparations (such as triamcinolone 0.1% solution) are used.
A 22-year-old man presents with a 3-month history of worsening diarrhea that comes and goes. While performing a comprehensive oral exam, you note two lesions on the buccal mucosa on the right side of the oral cavity. You document these lesions as two round lesions approximately 2 mm in diameter with a white-yellow center surrounded by a red halo. Pertinent positives also include a reduced appetite, abdominal pain, and cramping. Question - What is the most likely diagnosis? Answer Choices 1 Ulcerative colitis 2 Cholecystitis 3 Crohn's disease 4 GERD 5 Colon cancer
Correct Answer: Crohn's disease - This patient has an aphthous ulcer (canker sore, aphthous stomatitis). These painful open sores are found in the oral cavity and are the most common form of mouth ulcer. They are described as having a white or yellow center surrounded by a bright red area. They are benign, non-cancerous, and non-infectious, and the cause is often unknown, but aphthous lesions are common extraintestinal manifestations found in patients with Crohn's disease. Patients with ulcerative colitis (the other form of inflammatory bowel disease) may have intermittent diarrhea but do not typically develop aphthous ulcers. Common signs of cholecystitis include right-sided abdominal pain, vomiting, fever, positive Murphy's sign, and palpable gallbladder. GERD symptoms generally surround heartburn, although it can cause issues such as asthma, chronic cough, chronic laryngitis, sore throat, and non-cardiac chest pain. Colon cancer can present with anemic qualities on serology tests, abdominal pain, changes in bowel habits, and hemoccult-positive stool.
A 32-year-old woman presents with a 3-day history of irritation, burning, itching, and redness of both eyelids. She denies fever, visual changes, and photophobia. On physical examination, you note the presence of scales clinging to the eyelids bilaterally. Question - What is the proper management in this case? Answer Choices 1 Daily cleaning with a damp cotton applicator and baby shampoo 2 Short-term oral antibiotic therapy for 7 days 3 Short-term oral corticosteroid therapy for 14 days 4 Topical corticosteroid eye drops for 10 days 5 Prompt ophthalmologist referral
Correct Answer: Daily cleaning with a damp cotton applicator and baby shampoo - The scenario presented above depicts a patient with anterior blepharitis, which is a common disorder seen in primary care; it typically consists of a recurrent bilateral inflammation of the lid margins that involves the eyelid skin, eyelashes, and associated glands. Commonly, the underlying cause is seborrhea, which usually originates in the scalp, eyebrows, or ears. Sometimes, anterior blepharitis can be ulcerative, and the origin in the presented case is staphylococci. Anterior blepharitis can typically be resolved and controlled by cleaning the affected areas daily using a damp cotton applicator, warm water, and a baby shampoo mixture. The object of the daily cleaning is to remove the visible scales as efficiently as possible. None of the other listed options are an appropriate treatment plan for anterior blepharitis. Patients can also be diagnosed with what is known as posterior blepharitis, which is an inflammation of the meibomian glands of the eyes. It is usually staphylococcal in origin, and it typically presents with significantly worse signs and symptoms, such as hyperemic lids, the presence of telangiectasias, inflammation of the gland or their orifices, or even abnormal secretions; tears may be described as being frothy or greasy. More significant cases of posterior blepharitis can lead to conjunctivitis, hordeola, chalazions, eyelash trichiasis, or even corneal vascularization and thinning. Treatments for posterior blepharitis may consist of long-term oral antibiotic therapy, short-term topical steroids, or short-term topical antibiotics eye drops; if significant complications are evident, an ophthalmologist referral is indicated.
An 11-year-old girl with no significant past medical history presents with an increase in the frequency of her nosebleeds. She has experienced 3 episodes in the last week. Her mother was able to control and stop the bleeding by applying direct pressure to her external nasal area. Past medical history shows no significant issues with bleeding discrepancies or coagulopathies; she had a tonsillectomy when she was 8 years old; there were not any postoperative complications. Physical examination reveals a raw and irritated anterior right nare; the presence of dried crusted blood is noted. Question - What is the most likely cause of this patient's anterior epistaxis? Answer Choices 1 Digital trauma 2 Deviated septum 3 Genetic clotting disorder 4 Atherosclerotic disease 5 Hypertension
Correct Answer: Digital trauma - Anterior epistaxis is an extremely common presenting complaint in pediatric patients in a primary care office. Common factors are usually due to local trauma that predisposes patients to developing anterior epistaxis; they include (but are not limited to) nose digital trauma (nose picking), forceful nose-blowing/rubbing, administration of supplemental nasal oxygen, or low humidity. A deviated nasal septum can also predispose a patient to epistaxis, but it was not present on this patient's physical examination. The most likely source of this patient's epistaxis is Kiesselbach's plexus, which is a superficial venous plexus in the anterior septum. This is further supported by the physical exam findings of dried blood being seen in only one of the nares of the nose. Treatment of anterior epistaxis is usually direct pressure by compression at the site of the nares for at least 15 minutes; the patient should be in the sitting position and leaning slightly forward in order to decrease the swallowing of blood. A clotting disorder (or coagulopathy) can predispose a patient to developing epistaxis, but it is only found to be the underlying cause 5% of the time. Hypertension is rarely the cause of epistaxis in the pediatric population.
A 4-year-old Caucasian boy is seen for a 2-week history of purulent nasal discharge. He has been afebrile and has had no respiratory symptoms. Past history is unremarkable except for his mother's assessment that "he gets into everything." Examination revealed only a right-sided purulent nasal discharge, which was greenish-brown in color and extraordinarily foul-smelling. Question - What is the most direct method of diagnosing and treating this child's likely condition? Answer Choices 1 Culture and sensitivity of the nasal discharge 2 Direct visualization of the right nasal vestibule 3 Gram stain and KOH prep of the nasal discharge 4 Sinus radiographs 5 Soft tissue lateral radiograph of the nasopharynx
Correct Answer: Direct visualization of the right nasal vestibule - The history of an extremely foul-smelling unilateral purulent nasal discharge without other symptoms in an active young child strongly suggests a retained nasal foreign body, most readily diagnosed via direct visualization. Most foreign bodies can be removed during direct visualization or with positive pressure techniques. Culture and sensitivity and gram stain and KOH prep are not indicated, as the patient is afebrile and symptoms are not secondary to an infectious etiology. This will not treat the underlying cause of this patient's symptoms. Sinus radiographs will not visualize or remove the nasal foreign body. Soft tissue lateral radiographs may not be helpful, as many foreign bodies are radiolucent, such as vegetable foreign bodies. This will not treat the present condition.
You are evaluating a 24-year-old woman for bilateral eye pain. She describes red itching irritated eyelids for several weeks. She states she has had "several bouts" of similar symptoms over the last few years. Exam is consistent with blepharitis. She does not wear contacts, and she occasionally wears eye makeup. She denies any other infectious complaints. Question - What is the most appropriate treatment? Answer Choices 1 Start the patient on ophthalmic antibiotics. 2 Start the patient on ophthalmic steroids. 3 Discuss good eye hygiene and eyelid scrubbing. 4 Refer to an ophthalmologist for surgical correction. 5 Apply beta blocker to decrease intraocular pressure.
Correct Answer: Discuss good eye hygiene and eyelid scrubbing. - The correct answer is that you should discuss good eye hygiene and eyelid scrubbing with the patient. - Blepharitis is chronic inflammation of the eyelids. Patients often present with irritation of the lid margins that may produce minor crusting, a gritty sensation when blinking, eye itching and redness but a full preservation of vision. The majority of patients have recurrent bouts of inflammation, which can be resolved by eyelid washing and avoidance of eye makeup. The treatment of blepharitis rarely requires antibiotics. While it is possible to have a secondary infection—typically staph aureus—there is no evidence of a secondary infection in our patient. Initiation of ophthalmic steroids should be done under the guidance of an ophthalmologist. Additionally, there is no benefit of an ophthalmic steroid in the treatment of blepharitis. The majority of cases of uncomplicated blepharitis do not need to be referred to an ophthalmologist unless the diagnosis is uncertain or there are other complicating factors. Additionally, there is no surgical correction for uncomplicated blepharitis. Our patient does not have signs and symptoms of increased intraocular pressure.
A 21-year-old patient reports symptoms of sneezing, runny nose, itchy nose and eyes, and occasional intermittent cough. The patient's only brother has asthma and eczema. The patient has occasional headaches and there are dark circles under both eyes. Vitals are unremarkable. On exam, there is erythematous injection of the conjunctiva and pale boggy nasal mucosa. The patient requests medication to treat these symptoms that will not affect their daily activities. Question - What is the most appropriate medication for this patient? Answer Choices 1 Cetirizine 2 Diphenhydramine 3 Fexofenadine 4 Hydroxyzine 5 Loratadine
Correct Answer: Fexofenadine - Fexofenadine is the least sedating antihistamine. In clinical trials, fexofenadine was found to produce no more drowsiness than placebo. Fexofenadine is the only antihistamine that carries no package insert warning regarding sedation, driving, or operating machinery. It is indicated for allergic rhinitis and chronic idiopathic urticaria. Diphenhydramine and hydroxyzine are first-generation antihistamines, which are non-selective and highly lipophilic, and they readily cross the blood-brain barrier. They have prominent adverse effects of sedation and drowsiness. Cetirizine and loratadine are less sedating than first-generation antihistamines, but they both carry warnings regarding drowsiness and sedation, unlike fexofenadine.
4-year-old boy presents with a 4-day history of frequent sneezing, noticeable congestion, nasal discharge, and irritability; his mother has a history of airborne seasonal allergies and believes her son may have a problem with his sinuses. The mother denies noticing any shortness of breath, problems breathing, or fever; the boy has not been in close contact with sick individuals in the last few weeks. Physical examination reveals unilateral purulent nasal drainage from the left nare and a foul odor. Question - What is the most likely diagnosis? Answer Choices 1 Allergic rhinitis 2 Bacterial rhinosinusitis 3 Foreign body 4 Nasal polyp 5 Viral rhinitis
Correct Answer: Foreign body - The most likely cause of this patient's signs and symptoms is a foreign body. Pediatric patients and adult patients with psychiatric or behavioral problems are notorious for sticking objects in orifices and places they do not belong (e.g., nose, ears, and mouth). Common objects include beads, paper, rocks, toy parts, and organic material (e.g., peas, corn, seeds, nuts, legumes). Sometimes the patient is witnessed inserting the item, but other times the history and the physical examination together will reveal if there is a foreign body. The presence of unilateral purulent nasal drainage and a foul odor should create a high index of suspicion of a foreign body, which can also create symptoms such as sneezing, epistaxis, pain, and irritability. A cooperative patient is critical in terms of visualizing and removing any foreign body. With allergic rhinitis, a patient will develop nasal congestion, sneezing, rhinorrhea, and itchy nose/palate/throat/eyes; it can potentially be related to airborne seasonal or perennial allergens. Bacterial rhinosinusitis is a possibility, but the duration of the presenting problem is the main component ruling out this diagnosis. This diagnosis is typically made if a child seems to have viral rhinitis (common cold) that lasts longer than 10 days or if it worsens significantly after 5-7 days. Symptoms include nasal congestion, drainage, postnasal drainage, facial pain, headache, and fever. Nasal polyps are benign nasal tumors commonly seen in patients with allergic rhinitis. These can result in chronic nasal obstruction and diminished sense of smell, both of which are not consistent with this patient's history and physical examination. Viral rhinitis (common cold) is also a possibility, but patients will experience clear or mucoid rhinorrhea, not purulent drainage or foul odor. It is possible to also have nasal congestion, sore throat, or even a fever (childen under 6). Both nares will be affected.
A 2-year-old girl is brought to an otolaryngologist by her mother for chronic ear infections. The patient is otherwise healthy, with the exception of recurrent episodes of otitis media (OM). Examination and history show that the child has had average growth and development; she has not had invasive infections, skin disorders, or hospitalization. The child's mother is concerned about the risk of hearing loss and its effects on development. Question - What patient education information can you give the child's mother? Answer Choices 1 Hearing loss does not result from OM, except in rare cases. 2 Sensorineural hearing loss, not conductive, is associated with OM. 3 Hearing loss during OM may adversely affect cognition and language. 4 Van der Hoeve syndrome, a sequela of OM, can cause hearing loss. 5 Hearing loss associated with OM is always conductive and temporary.
Correct Answer: Hearing loss during OM may adversely affect cognition and language. Conductive and sensorineural hearing loss are complications of chronic otitis media (OM). Acute and chronic suppurative OM usually results in conductive hearing loss. Chronic infection may result in conductive hearing loss from a perforation of the tympanic membrane. Sensorineural hearing loss can occur, especially when herpes zoster is the etiologic agent. Van der Hoeve syndrome is a constellation of symptoms including hearing loss, but is unrelated to OM. Hearing impairment is a risk factor for impaired speech and language development, particularly if it occurs early in life. In cases where hearing loss due to chronic OM is reversed surgically, it is likely that young children will compensate and catch up to peers. In cases where OM is either undiagnosed or untreated, long-term developmental and social problems may result. Because otitis media is often associated with hearing loss, most clinicians have been eager to treat the condition to restore hearing to normal, preventing any long-term problems.
68-year-old man with a 50 pack-year smoking history presents to his primary care provider due to a 4-month-history of progressive dysphagia. His review of symptoms is notable for intermittent ear pain (especially upon swallowing), an involuntary weight loss of 12 lb over the past 4 weeks, and occasional hemoptysis. He denies chills, abdominal pain, shortness of breath, chest pain, vomiting, or skin changes. His physical exam is remarkable for non-tender immobile lymphadenopathy of the cervical nodes and stridor upon auscultation of the trachea. A nasopharyngoscope revealed complete replacement of the right true cord with a mass lesion. Question - What additional finding is most likely in this patient? Answer Choices 1 Weight gain 2 Tonsillar exudates 3 Hoarseness 4 Erythematous tympanic membrane 5 Fever and retropharyngeal edema
Correct Answer: Hoarseness - This patient demonstrates signs and symptoms consistent with laryngeal cancer. A change in voice quality is most often the presenting complaint; new and persistent hoarseness is a common finding. Squamous cell carcinoma of the larynx, the most common malignancy of the larynx, occurs almost exclusively in patients with a history of significant tobacco use. Other manifestations may occur, including persistent throat or ear pain, especially with swallowing, hemoptysis, lymphadenopathy, a neck mass, dysphagia, weight loss, and airway compromise with stridor. - Weight gain is unlikely with laryngeal cancer, and the patient notes losing 12 lb recently. - Tonsillar exudates and tympanic membrane abnormalities are not expected in laryngeal carcinoma; they suggest infectious etiologies. - Fever and posterior pharyngeal edema are more commonly associated with a retropharyngeal abscess; this process is infectious and of an acute progression.
A 28-year-old male patient presented 4 days ago due to a 101.8°F fever and received a diagnosis of acute pharyngitis. The provider prescribed penicillin VK 250 mg TID for 10 days. The patient returns today because the sore throat has worsened. He has been unable to drink fluids or eat, and he has excruciatingly severe pain when swallowing. He speaks with a muffled "hot potato" voice. Upon re-examination today, there is a right medial deviation of the soft palate with 4+ right tonsillar swelling. Question Based on the most likely diagnosis, what is the most appropriate next step in treatment? Answer Choices 1 Intravenous antibiotics 2 Incision and drainage 3 Intravenous antivirals 4 Observation and reassurance 5 Immediate tonsillectomy
Correct Answer: Incision and drainage - The patient scenario above is describing a classic peritonsillar abscess. This occurs when the acute infection of the pharyngeal area penetrates the tonsillar capsule and then invades the surrounding tissue. This is potentially a complication from acute bacterial pharyngeal infections, which can include streptococcus pharyngitis. These patients will describe a severe sore throat, odynophagia, trismus, and upon examination will be noted to have a medial deviation of the soft palate and peritonsillar fold, as well as have an abnormally muffled voice, often referred to as a "hot potato" voice. - The correct response is aspiration via incision and drainage (I&D) or needle aspiration. This provides immediate symptoms improvement. The abscess is localized and blunt dissection is done to break loculations. The incision it is left open to drain. After cultures are obtained from the abscess, antibiotic therapy should begin. Because of the severity of symptoms this patient is presenting with, the treatment for this condition at this point usually will consist of intravenous amoxicillin (1 g), amoxicillin-sublactam (3 g), or clindamycin (600 - 900 mg). Intravenous antivirals would not be an appropriate choice, as the organisms that are more likely to cause this include aerobic (Streptococcus pyogenes, Staphylococcus aureus, Haemophilus influenzae, Neisseria species) and anaerobic (Fusobacterium, Peptostreptococcus, Prevotell, and Bacteroides) organisms. Observation and reassurance is not appropriate due to the substantial signs and symptoms the patient is currently displaying. Immediate tonsillectomy is not a recommended plan of action for acute treatment of a peritonsillar abscess. Experts have concluded that it is safest to complete a tonsillectomy approximately 3 to 6 months after the abscess has resolved in patients who have either recurrent tonsillitis or peritonsillar abscess. There is no indication of this being a recurrent event in this patient.
25-year-old woman presents due to sneezing episodes that have progressively worsened over the last few months. She has symptoms on most days, and it is affecting her daily life. The symptoms are worse in the spring and fall and improve in the winter. She also reports rhinorrhea, cough, and fatigue. Physical examination is remarkable for boggy nasal mucosa and pale nasal turbinates. Question - What is the most appropriate first-line clinical intervention? Answer Choices 1 Inhaled corticosteroids 2 Oral decongestants 3 Intranasal decongestants 4 Intranasal corticosteroids 5 Oral steroids
Correct Answer: Intranasal corticosteroids - This patient has allergic rhinitis, an IgE-mediated response to allergen exposure in the nasal passages. Patients present with sneezing, rhinorrhea, and itching of the eyes and nose. The most appropriate first-line treatments are intranasal corticosteroids (ICS), antihistamine nasal sprays, and oral antihistamines. ICS are considered the most effective therapy in patients with moderate to severe symptoms. Inhaled corticosteroids are used in patients with asthma, but not in patients with allergic rhinitis. Oral steroids and oral decongestants are not indicated for initial treatment of allergic rhinitis. Decongestants provide no relief and are associated with side effects such as hypertension and insomnia. Oral steroids do provide some relief of symptoms, but they are not indicated for long-term therapy because of the significant side effect profile. Intranasal decongestants should not be prescribed to patients with allergic rhinitis. These over-the-counter nasal sprays will lead to rhinitis medicamentosa after 5-7 days of treatment; a characteristic of this condition is severe rebound nasal congestion.
A 42-year-old man presents with a 10-day history of worsening headache, stuffy nose, greenish nasal discharge, and a low grade fever. He has body aches and facial pain, as well as a dry cough. He denies shortness of breath, abdominal pain, nausea, or vomiting. He is a non-smoker, has no significant past medical history, and is only taking acetaminophen. On exam, he has a temperature of 100.9°F taken orally. Pulse is 86/min, BP is 120/76 mm Hg left arm sitting, and SPO2 is 94% on room air. Lungs are clear and abdomen normal. Nasal mucosa appears boggy, and there is tenderness with palpation over the facial bones (maxillary area). Pharynx is without exudates. Question - What component of the history is the most informs your decision to prescribe antibiotics for this patient? Answer Choices 1 Facial pain, body aches, and a SPO2 of 94% 2 Length of time the symptoms have been present 3 Boggy nasal mucosal with facial tenderness 4 Low-grade fever 5 Headache and body aches
Correct Answer: Length of time the symptoms have been present - The condition described in this clinical scenario is highly indicative of acute bacterial rhinosinusitis (acute sinusitis). Acute sinusitis clinically is described as including symptoms such as green/yellow purulent appearing discharge, nasal obstruction, congestion, facial pain, or pressure over the affected sinus, and may also include cough, malaise, fever, or even headache. Acute sinusitis has an acute onset of symptoms, ranging from 1-4 weeks in length of duration by the time the patient presents clinically. More commonly, the origin of sinusitis is viral, but symptoms relating to this will resolve as time passes, not intensify or worsen. Bacterial sinusitis can be distinguished from viral sinusitis by symptoms persisting or worsening beyond 10 days duration. Otherwise, the symptoms described in this case and with the other answer choices can be present in both viral and bacterial sinusitis.
A 62-year-old female patient with a history of breast cancer and rheumatoid arthritis presents with a stiff neck and severe headache that started a couple of days ago; symptoms are worsening. Upon waking this morning, she felt nauseated and vomited twice. Neurological examination shows right-sided hemiparesis and dilated and non-reactive left pupil. Question - What is the most appropriate next step? Answer Choices 1 Magnetic resonance imaging 2 Lumbar puncture 3 Perform doll's eye maneuver 4 Arrange for a brain biopsy 5 Chest x-ray
Correct Answer: Magnetic resonance imaging - A history of breast cancer and signs of possible intracranial hypertension suggest a metastatic tumor in the brain. Right-sided hemiparesis and dilated non-reactive pupil on the left point to the compression from the left side. While in the office, the patient should be examined for the presence of papilledema. Papilledema will confirm the suspicion of a transtentorial herniation, where brain tissue bulges out of the cranium through the tentorial notch due to increased intracranial pressure. Magnetic resonance imaging (MRI) of the endocranium must be scheduled to make a definitive diagnosis. Brain biopsy is only necessary if the diagnosis is in question. It is not an appropriate next step in a symptomatic patient. Lumbar puncture is contraindicated in focal lesions, particularly when transtentorial herniation is suspected; by taking a sample of cerebrospinal fluid, brain tissue can press vital centers in the brainstem and cause the death of the patient. Doll's eye maneuver is a test of brainstem integrity performed in comatose patients. The patient's head is quickly moved to one side and then to the other; if the brainstem is intact, the eyes will move conjugately away from the direction of turning and revert back to the midline. Based on history of rheumatoid arthritis and a stiff neck, this patient may have atlantoaxial subluxation; moving the neck can cause serious damage to the cord. This patient may have metastases in the lungs and chest x-ray may be considered, but these symptoms are of intracranial hypertension, not a process in the lungs.
A 23-year-old man presents with a 2-day history of watery nasal discharge, malaise, sneezing, and nasal congestion. On examination, you notice inflammation of the nasal mucosa; pulse is 80/min, BP is 130/84 mm Hg; temperature is 98.8°F. The rest of the examination is normal. Question - What is the appropriate treatment for this patient? Answer Choices 1 Azithromycin, nasal decongestants, ibuprofen 2 Ibuprofen only 3 Nasal decongestants and non-pharmacologic remedies 4 Antiviral agents, nasal decongestants, ibuprofen 5 Antiviral agents only
Correct Answer: Nasal decongestants and non-pharmacologic remedies - This patient most likely has acute rhinitis, which is treated with nasal decongestants and steam inhalation. Ibuprofen can be added in cases of fever but is unnecessary in this case, as the patient is afebrile. Azithromycin is a macrolide antibiotic; its use for treating acute rhinitis is incorrect. Antibiotics have no role in the treatment of uncomplicated non-specific upper respiratory infections; their misuse facilitates the emergence of antibiotic resistance. Only symptomatic treatment with decongestants and steam inhalation is necessary for treating acute rhinitis. Antiviral agents are used in cases of an immunocompromised host and are not essential for otherwise healthy patients.
A 52-year-old man presents with a concern of hearing changes. He has noticed a decreased ability to hear sounds for the past few months; he tested it at home by covering each ear, and he now thinks there is a hearing loss in only the left side. Furthermore, he hears a ringing sound all the time. He is a business manager, and he denies occupational exposure to loud noises. He denies head trauma, headaches, and prior ear problems. His wife thinks this is just normal age-related hearing loss. His review of systems is negative for other neurological symptoms. The patient's past medical history is unremarkable; he has no known medical conditions. He takes no medications. He has no allergies, and he has not had any surgeries. He denies alcohol, tobacco, and drug use. On physical exam, his vitals are normal. His HEENT exam is significant only for decreased auditory acuity and Weber test lateralizing to the right. Audiometry confirms a sensorineural hearing loss on the left. An MRI is performed; it shows a well-delineated intracranial mass. Further investigation reveals the origin of cells is from Schwann cells. Question - What choice represents the best intervention for this patient's curren
Correct Answer: Referral for surgery - This patient is presenting with a vestibular schwannoma, or acoustic neuroma, that is affecting his vestibulocochlear nerve (cranial nerve VIII). This is one of the more common benign head and neck neoplasms. A common presentation is unilateral hearing loss and tinnitus. Treatment is typically surgical removal; another possibility is radiation therapy. Of the choices listed, referral for surgery is the best option; if he turns out to be a poor surgical candidate, radiotherapy should be discussed. This patient should not be referred for chemotherapy. Schwannomas are not typically responsive to chemotherapy, so doing so could delay appropriate treatment. If the patient was describing seizures, the provider should refer for electroencephalography (EEG), especially as some brain tumors are associated with seizures. However, this patient denied other neurological symptoms and has been clearly shown to have an intracranial mass. Referral for EEG would also delay definitive treatment. A referral for unilateral hearing aid would not fix this man's hearing loss; the treatment of his brain tumor would be delayed. A referral for ventricular shunt should be done for patients with hydrocephalus. Shunts have no role in schwannoma treatment.
A 73-year-old man presents with a nosebleed that will not stop. The bleeding has been present for over 2 hours. The patient's nose began dripping blood at breakfast; there was no known trauma. He denies pain. He has tried applying nasal pressure and lying down to rest. Until the bleeding began, the patient had not been experiencing any nasal symptoms, such as congestion, impaired nasal patency, or rhinitis. Because he felt too dizzy to drive, his wife drove him to the emergency department. The patient has no diagnosed medical conditions and takes no medications. He denies prior episodes of severe nosebleeds, easy bruising, and any known bleeding disorder. The patient's vitals are shown in the table. 148lb, 69", P-120, RR18, BP 90/66, T-97.4f/36.3c On physical exam, the man is holding a large bloodsoaked towel to his nose, with continued brisk bleeding. He is otherwise in no apparent distress but seems somewhat confused. On rhinoscopy, bleeding is observed from bilateral nares. No foreign body, mass, lesions or abrasions are visualized. The bleeding site cannot be identified. Question - What intervention is the next step for this patient's current condition? Answer Choices 1 Apply elec
Correct Answer: Obtain IV access and begin normal saline infusion. - This patient most likely has a posterior epistaxis. Posterior bleeds are less common than anterior bleeds, but they should be suspected when bleeding is high in volume and unresponsive to nasal packing/tamponade. The cause of this patient's posterior bleed is not evident; many are idiopathic. With confusion, tachycardia, tachypnea, hypotension, and blood loss, however, the provider must recognize that this patient is presenting with (hypovolemic) shock, which must be addressed before the epistaxis. Healthcare personnel should obtain IV access and begin normal saline infusion. If a patient presents with shock and obvious blood loss, first-line actions include control of bleeding, crossmatching of blood, and infusion of fluids and blood products. When a patient is hemodynamically stable and the provider can visually identify a blood vessel site for epistaxis, it is reasonable to apply electrocautery to visible nasal blood vessels. It is common practice to apply nasal packing when bleeding points cannot be identified and an otolaryngologist is not available. Anterior packing alone or a posterior balloon plus anterior packing may stabilize the bleed. Nasal and mucosa trauma can occur with packing. Packing can be performed once basic shock measures are completed. If this patient was hemodynamically stable and a trained surgeon was available, it would be reasonable to refer for surgical ligation of the nasal arterial supply. Vessel ligation success rate is quite high, but this hypotensive patient would not tolerate surgery. For some nasal procedures, the provider may spray 4% topical cocaine intranasally for its decongestant and anesthetic effects. This may be used prior to electrocautery. This intervention is inappropriate before basic shock measures are completed.
A 42-year-old male patient presents for evaluation of a growth on the tongue. He thinks that the lesion has been present for a few months, and it has not changed, but he generally prefers to avoid healthcare, and he has not been concerned. He is only here at the urging of a family member. The patient denies oral symptoms and changes in taste sensation; he states that he generally feels fine. The patient denies using chewing tobacco or cigarettes. On physical exam, there is a white patch of tissue that does not scrape off; there is a "shaggy" appearance on the left lateral tongue. No erythema is noted. No other lesions are identified. The exam is otherwise unremarkable. A biopsy of the lesion is obtained. The pathology shows hyperkeratosis, "balloon" cells in the upper cell layer, and Epstein-Barr virus (EBV) in the basal epithelial cells. Question - What is the most likely diagnosis? Answer Choices 1 Geographic tongue 2 Oral candidiasis 3 Oral hairy leukoplakia 4 Oral lichen planus 5 Squamous cell carcinoma
Correct Answer: Oral hairy leukoplakia - This patient has oral hairy leukoplakia, often associated with HIV infection or other immunocompromised states (eg, post-transplant). It is rare in immunocompetent patients. On exam, the lesions are white plaques that cannot be scraped off (unlike candidiasis). The oral hairy leukoplakia is a benign neoplasm of the tongue, not of great significance in itself, but due to its association with an immunocompromised state, it can be a harbinger of a more significant condition. Geographic tongue is a benign condition of the tongue; it affects the epithelium. It typically presents with erythematous patches, with white rounded borders. Lesions can change size, pattern, and location. Patients may be asymptomatic or may experience burning or oral discomfort. Oral candidiasis (thrush) is a fungal infection of the oral mucosa. Affected patients typically note oral pain or discomfort. There may be white plaque with surrounding or underlying erythematous tissue. Lesions can be wiped or scraped off. Biopsy is not typically necessary for diagnosis; this patient's biopsy would indicate fungal organisms if he had thrush. Oral lichen planus can present with white "lacelike" patches, but it typically presents with oral pain. Squamous cell carcinoma can present as various lesions (plaques, ulcerations, erosions, papules). Any persistent oral lesion should be biopsied to evaluate for possible malignancy. This patient's biopsy does not indicate malignancy.
39-year-old Caucasian man presents with a "lump in his left ear canal." He just wants to confirm it is not a type of tumor. Otoscopic examination reveals a single discrete pedunculated flesh-colored bony mass located at the 7 o'clock position in the left external auditory canal. The right external ear canal was unremarkable. You tell the patient that the lump in his ear canal is indeed classified as a tumor but is completely benign. Question - What is this lump called? Answer Choices 1 Keratoacanthoma 2 Osteoma 3 Pilomatrixoma 4 Sebaceous adenoma 5 Squamous papilloma
Correct Answer: Osteoma - Osteoma will manifest as discrete pedunculated bony masses that arise in the external solitary canal, usually from the tympanosquamous suture line. Osteomas are typically solitary, unilateral, and slow-growing. They are also the most common bony neoplasms of the temporal bone. Usually osteomas are asymptomatic, and no treatment is necessary. They can cause issues, however, if the canal becomes obstructed. This lesion is also commonly referred to as surfer's ear due to the possibility of chronic exposure to cold water/wind leading to their formation. Keratoacanthoma is a lesion linked to sun exposure, chemical carcinogens, trauma, or a viral etiology. These are located more often on the skin of the face, forearms, and dorsal aspects of the hands. Actinic damage is evident in the surrounding skin around the lesion. They can be smooth firm symmetric erythematous lesions or skin-colored lesions with a central keratotic area. They can also be centrally necrotic, depending on the stage of the lesion. Characteristics of pilomatrixoma include necrosis, keratinization, and proliferation of small epithelial cells; calcification and ossification may also be evident. These are slow-growing bluish solitary well-circumscribed nodules, usually in the dermis of the auricle or preauricular area. Sebaceous adenomas are extremely rare lesioms. They arise from the sebaceous glands and they present as smooth elevated pedunculated firm-to-soft lesions that are solitary and smaller than 0.5 cm. They can be tan, skin-colored, pink, or red. Squamous papilloma is another benign external auditory canal tumor, but these will present as wary verrucous elevated lesions that are pigmented and have an overlying thickened keratin layer.
A 12-year-old Caucasian girl presents with a sore throat. The onset of symptoms was about 24 hours ago. The patient experiences pain in her throat, especially with talking or swallowing. She is fatigued because throat pain prevented her from sleeping last night. Throat lozenges have not been helpful. Several classmates have been out sick recently. She denies nasal congestion, rhinorrhea, and cough and is unsure of fevers. This patient has no chronic medical conditions, takes no medications, and has no known drug allergies. On physical exam, she appears slightly ill and fatigued. HEENT exam is positive for bilateral cervical lymphadenopathy and inflamed posterior oropharynx without exudate. She does have normal range of motion of the neck without eliciting pain. Her heart, lung, and abdominal exams are normal. No other lymph nodes are palpable. Weight 92 lb, height 4'8", pulse 95, BP 102/60, temperature 99.2°F/37.3°C. Question - What is the most appropriate next step in the care of this patient? Answer Choices 1 Excuse the patient from school until symptoms have fully resolved. 2 Initiate amoxicillin. 3 Order complete blood count (CBC) with differential. 4 Perform rapid antigen test
Correct Answer: Perform rapid antigen testing for group A streptococcus. - The most appropriate step in the management of acute pharyngitis is rapid antigen testing for group A streptococcus. Viral causes of pharyngitis are common, but management should focus on identifying those at risk for group A beta-hemolytic streptococcal infections to prevent compilations like rheumatic fever and glomerulonephritis. This patient presents with acute pharyngitis, no cough, and cervical lymphadenopathy—a likely diagnosis of streptococcal origin. Additional indicators would be if the patient's temperature were >38°C and if there were tonsillar exudate present. Excusing the patient from school may limit spread of infection and allows for rest, but it does not address the possible need for antibacterial treatment and leaves the patient at risk for developing complications if she has group A streptococcal pharyngitis. Initiation of amoxicillin may be appropriate once group A streptococcal pharyngitis is confirmed (penicillin has been the standard primary regiment). If this patient's pharyngitis is not caused by Streptococcus, however, antibiotics are unnecessary and put the patient at risk for adverse effects. If this patient's pharyngitis is caused by infectious mononucleosis, amoxicillin can precipitate a skin rash. Complete blood count (CBC) will likely show characteristic changes in patients with acute pharyngitis. Depending on the potential cause of the pharyngitis, neutrophils and monocytes may be elevated for bacterial and viral causes, respectively. CBC changes are unspecific, however, especially early in infection. It is more sensible to swab the throat directly and get a result within minutes rather than to subject a child to a blood draw with a longer wait time. The conservative approach of fluid, rest, and analgesics/antipyretics is reasonable if the patient is not likely to have group A streptococcal pharyngitis, but the link between aspirin and Reye's syndrome in children discourages its use, especially in viral illnesses.
17-year-old boy was in your clinic 4 days ago for evaluation of a 101.8°F fever and was diagnosed with acute pharyngitis. You prescribed penicillin VK 250 mg TID for 10 days. The patient returns today because his sore throat is now worse. He has not been able to drink fluids and he has excruciatingly severe pain with swallowing. You recognize the muffled "hot potato" voice. On re-examination, you identify a right medial deviation of the soft palate with a 4+ right tonsillar swelling. Question What is the most likely diagnosis? Answer Choices 1 Oral candidiasis 2 Peritonsillar abscess 3 Laryngitis 4 Mononucleosis 5 Dental abscess
Correct Answer: Peritonsillar abscess - This patient is clearly suffering from a peritonsillar abscess. This occurs when an active infection penetrates the tonsillar capsule and then involves the surrounding tissue. These patients will have a severe sore throat, odynophagia, trismus, deviation of the soft palate, and an abnormally muffled voice (hot potato voice). Oral candidiasis (or thrush) does not present with the symptoms described in the scenario. Typically, oral candidiasis is painful and appears as creamy-white, curd-like patches; overlying erythematous mucosa can be found virtually anywhere in the oral cavity. The white patches can easily be wiped off when attempted. Laryngitis is lower on the differential diagnosis list because it usually presents with the primary symptom of hoarseness. Laryngitis frequently occurs approximately 1 week after the occurrence of an upper respiratory viral infection that has since resolved. A dental abscess would cause severe, persistent, throbbing toothache, sensitivity to hot and cold temperatures, sensitivity to biting or chewing, fever, possibly noticeable swelling in the face or cheek, or even lymphadenopathy relating back to the site of the abscess. The symptoms of a dental abscess do not match the clinical scenario presented. Mononucleosis also presents somewhat differently from the scenario above, making it a less likely diagnosis. Malaise, fever, sore throat (sometimes exudative), lymphadenopathy, palatal petechiae, and even splenomegaly are found in patients with mononucleosis.
A 19-year-old woman presents with a painful sore in her mouth that has been present for 3 days. She denies oral trauma. She describes a single painful lesion that feels like a bump between her cheek and gum line. She has not tried any treatments at home. She denies history of similar lesions. She has otherwise been feeling well and denies recent flu-like symptoms. Her past medical history is unremarkable, with no known medical conditions or history of surgery; she takes no medications and has no allergies. She lives in an apartment with her boyfriend; she works at a convenience store; and she denies the use of alcohol, tobacco (including chewing tobacco), and recreational drugs. On physical exam, her vitals are normal. A single tender lesion is identified on the buccal mucosa, just across from the lower right jaw. It is a small shallow ulcer approximately 3 mm diameter with a yellow-gray center surrounded by a red halo. The remainder of her exam is normal. Question - What is the best intervention for this patient's condition? Answer Choices 1 Prescribe oral antiviral. 2 Prescribe topical corticosteroids. 3 Refer to a dermatologist. 4 Refer to an oral surgeon. 5 Test patient for aut
Correct Answer: Prescribe topical corticosteroids. -This patient presents with an aphthous ulcer (canker sore), a very common oral lesion. Treatment is targeted at pain relief. Topical corticosteroids have been shown to provide symptomatic relief. Aphthous ulcers tend to resolve on their own within 10-14 days. An aphthous ulcer may be confused with herpes; an association with human herpesvirus 6 has been found. Aphthous ulcers are self-limiting, however, so prescribing oral antiviral medication is not recommended. This condition can be managed with any urgent care or family practice, so it does not require a referral to a dermatologist. Such referral would delay pain treatment and incur unnecessary healthcare costs. Aphthous ulcers are not malignant, and this presentation is not similar to any oral lesion that would require surgical removal, so she should not be referred to an oral surgeon. Recurrent aphthous ulcers can be linked with celiac disease, HIV, and inflammatory bowel conditions. It would be reasonable to test this patient for autoimmune diseases if she presented with recurrent ulcers. With a single occurrence, no further testing is recommended.
A 28-year-old woman with a past medical history of well-controlled asthma presents with recurrent sneezing episodes, nasal itching, congestion, and headache. Physical exam is remarkable for post-nasal drip, transverse nasal crease, and bilateral infraorbital cyanosis. An internal nasal speculum exam reveals edema and pale bluish nasal mucosa. Question - Given the most likely diagnosis, what diagnostic evaluation can be made about this patient? Answer Choices 1 Sensitivity to possible etiologies is determined with skin testing. 2 The diagnosis is supported by an elevated monocyte count. 3 Serum IgE level measurements are sensitive and specific for this diagnosis. 4 Radiographic imaging is necessary to confirm the suspected illness. 5 Eosinophils upon nasal cytology may be used as the sole confirmatory diagnostic finding.
Correct Answer: Sensitivity to possible etiologies is determined with skin testing. - Sensitivity to possible etiologies is determined with skin testing. In vivo allergy skin tests (immediate hypersensitivity testing) can determine immediate (IgE-mediated) hypersensitivity to specific allergens, including virtually all allergens that cause allergic rhinitis. This patient's medical history and current presentation are remarkable for allergic rhinitis. Signs and symptoms include sneezing paroxysms, nasal, ocular, or palatal itching, clear rhinorrhea, nasal congestion, pale, bluish nasal mucosa, transverse nasal crease, infraorbital cyanosis ("allergic shiners"), and serous otitis media. Total IgE level in patients with allergic rhinitis are more likely to be elevated than the general population, but this test is neither sensitive nor specific for allergic rhinitis. Up to 50% of patients with allergic rhinitis have reference levels of total IgE, while 20% of unaffected individuals can have elevated total IgE levels. - As is the case with total serum IgE, though it is not a sensitive or specific test, an elevated eosinophil count points toward allergic rhinitis. Radiographic studies are unnecessary in the diagnosis of allergic rhinitis, but they can be helpful for evaluating possible structural abnormalities or detecting complications or comorbid conditions, such as sinusitis or adenoid hypertrophy. The presence of eosinophils upon nasal cytology with a nasal smear is consistent with allergic rhinitis, but they can also be observed in NARES (non-allergic rhinitis with eosinophilia syndrome). Results are neither sensitive nor specific for allergic rhinitis, and the results should not be used exclusively for establishing the diagnosis.
A 22-year-old patient presents because they developed a fever of 101°F this morning. They have a 2-week history of rhinorrhea, congestion, and headache. The patient states that the rhinorrhea was initially clear and improved after 5 days, but then it returned and is now thick and greenish in color. The headache is felt in the forehead and cheeks, and it worsens when bending over. Question - What pathogen is most likely causing this patient's current symptoms? Answer Choices 1 Rhinovirus 2 Adenovirus 3 Streptococcus pneumoniae 4 Pseudomonas aeruginosa 5 Staphylococcus aureus
Correct Answer: Streptococcus pneumoniae - Symptoms of rhinorrhea accompanied by sinus pressure suggest sinusitis. Viruses cause most cases of sinusitis, but this presentation is more suggestive of a bacterial etiology for the following reasons: - Duration of symptoms is longer than 10 days. - Clinical condition improved then worsened again, suggesting that the patient may have initially had a virus that allowed for a secondary bacterial infection to develop. - Fever and purulent nasal discharge are present, both of which are more likely with bacterial sinusitis. Streptococcus pneumoniae is the most common causative agent. Other streptococci species and Haemophilus influenzae are also common causes of bacterial sinusitis.= Rhinovirus and adenovirus are more typical viral causes of sinusitis rather than the bacterial sinusitis suggested by this presentation. Pseudomonas aeruginosa is not a common cause of sinusitis. Staphylococcus aureus may cause sinusitis, but Streptococcus pneumoniae is more common.
A 48-year-old patient with hypertension presents with worsening hearing loss and tinnitus in the right ear. It began about 3 months ago and was initially bearable; it has progressed to hearing nothing in the right ear, and the tinnitus is unrelenting and constant. The patient is worried by new issues with balance and coordination. MRI reveals a 4 cm enhancing lesion of the right internal auditory canal. Question - What is a reasonable management option at this time? Answer Choices 1 Chemotherapy 2 Linear accelerator radiation therapy 3 Observation only 4 Proton beam therapy 5 Surgical resection
Correct Answer: Surgical resection - Explanation Vestibular schwannoma (acoustic neuroma) is one of the most common intracranial tumors encountered in clinical practice, most often occurring as unilateral lesions and rarely as bilateral masses. Growth of these lesions is overall slow, but the increased size can eventually cause unilateral hearing loss and deterioration of speech discrimination. Tinnitus will be seen in these patients as the tumor increases in size and as more central nervous system components are affected. This will cause loss of balance and coordination, vertigo, facial numbness, facial weakness, or even dysphagia. These lesions are typically diagnosed via MRI. Surgical resection via craniotomy is generally the preferred approach for large, symptomatic tumors. Chemotherapy is not used for treating schwannomas. Linear accelerator radiation therapy and proton beam therapy are types of radiotherapy used with increasing frequency in managing schwannomas. Radiotherapy is not considered first-line in patients with large tumors who are good surgical candidates. Observation only is not appropriate for this patient because significant symptoms are interfering with quality of life.
25-year-old female patient presents with a 2-day history of acute pain in the mouth; there is no significant past medical history. The patient states that the pain is in a localized part of the inner cheek; it is aggravated by eating food, speaking, and smiling. She denies any history of smoking, risky sexual behavior, drug use, transfusions, trauma, fever, chills, otalgia, otorrhea, rhinitis, eye pain/discharge, halitosis, regurgitation, skin changes, swollen glands, sore throat, or headache. Physical exam is noteworthy for painful ulcerations of buccal and labial mucosa with a yellowish base and red halos. Question - What is the most appropriate intervention? Answer Choices 1 Referral to head and neck surgeon for excision 2 Increased vitamin B, C, and D 3 Oral antibiotic therapy 4 Thalidomide 5 Topical corticosteroids
Correct Answer: Topical corticosteroids - Aphthous ulcers are common and easy to recognize. They are found on freely moving non-keratinized mucosa like buccal and labial mucosa, and they are not attached to gingiva or palate; they may be single or multiple. They are usually recurrent, and they appear as painful, small, round ulcerations with yellow-gray fibrinoid centers surrounded by red halos. Minor aphthous ulcers are less than 1 cm, and they generally heal in 10-14 days. Topical corticosteroids appear to provide symptomatic relief in many patients with aphthous ulcers. A referral to a head and neck surgeon is not necessary because there is no surgical intervention necessary for this diagnosis. Treatment or prevention of aphthous ulcers with vitamins has not proven beneficial. Treatment with antibiotics is not warranted; this is not a bacterial infection. Thalidomide has been used selectively in recurrent aphthous ulcerations in patients who are HIV-positive.
40-year-old man with a past medical history of hepatitis C presents with burning and pain of the oral cavity, associated with a pruritic rash of the flexor aspect of the left wrist. Physical exam is remarkable for violaceous, shiny, polygonal papules arranged as lines and circles on the wrist. The papules range from 1 mm to 1 cm in diameter and have fine white lines on them. In the oral cavity, a reticular, white, lacy pattern is visualized. Question - What is the most appropriate first-line pharmacological treatment for this patient's oral lesions? Answer Choices 1 Triamcinolone oral paste 2 Cyclosporine capsules 3 Acitretin 4 Nystatin oral suspension 5 Doxycycline oral suspension
Correct Answer: Triamcinolone oral paste - This patient has lichen planus with cutaneous and oral involvement. This inflammatory mucocutaneous condition usually exhibits a distinctive morphology and is associated with hepatitis C. The classic appearance of skin lesions includes violaceous polygonal flat-topped papules and plaques, commonly occurring at the wrist. Topical steroids are usually tried first for lichen planus of the oral mucosa. Treatment with steroids is especially indicated in ulcerative forms of oral lichen planus. Topical and systemic cyclosporin has been tried with some success, but a randomized double-blind study indicated that topical cyclosporin was a less effective and much more costly regimen than clobetasol. Newer topical calcineurin inhibitors have replaced topical cyclosporin for the treatment of lichen planus. Acitretin has been reported to be effective for the treatment of severe cutaneous disease; it is not used for oral lichen planus. Nystatin and doxycycline suspensions are not indicated, as this patient is not experiencing a fungal or bacterial infectious process.
52-year-old man presents for a follow-up visit. He saw you about 2 weeks ago due to losing his voice. The diagnosis of acute laryngitis was made at that time, and supportive treatment was described to him. The patient returns a little worried today because he has had no improvement in getting his voice back since his last visit. The patient is a professor at a local university and desperately needs his voice to return in order to lecture properly. Social history reveals a 32 pack-year smoking history. Further questioning of the patient during your review of systems reveals unintentional weight loss of 5 lb since his last visit. Question - What would be the next course of action based on the history and physical examination? Answer Choices 1 Observation for another 2 weeks 2 Reassurance and supportive treatment 3 Urgent referral to a pulmonologist 4 Urgent referral to an otolaryngologist 5 Initiation of a daily PPI and return for follow up
Correct Answer: Urgent referral to an otolaryngologist - Acute laryngitis is the most common cause of hoarseness in patients of all ages. Symptoms will commonly last a week or more even after the resolution of an upper respiratory infection. Treatment at that point includes avoidance of vigorous use of the voice (singing, shouting) until it returns to normal. Acute laryngitis is often viral in origin, but there has been incidence of M. catarrhalis and H. influenzae isolated from the nasopharynx. Antibiotics are occasionally prescribed to help with hoarseness and cough. A ≥2-week duration of hoarseness in heavy smokers, especially males ages 50-70, suggests squamous cell carcinoma of the larynx. Also, unintentional weight loss is a constitutional symptom/sign that cannot be missed. This patient needs referral to an otolaryngologist for further evaluation. Further observation and reassurance have an extremely low probability of bringing resolution to this urgent case. A pulmonology referral is not indicated, as he is not experiencing shortness of breath, and primary symptoms are from the larynx. Although the patient has the potential of having laryngopharyngeal reflux and needing a PPI, the pertinent positives at this point more towards a type of malignancy.
A 24-year-old man with no significant past medical history presents with a 3-month history of progressive hearing loss of his left ear. He states that he has the greatest difficulty in hearing high-pitched sounds. He denies trauma, recent travel, sick contacts, pressure changes, headache, nausea, otorrhea, otalgia, fever, chills, vertigo, swollen glands, rashes, sore throat, vision changes, or rhinitis. He states that he enjoys listening to music on his personal music device and attends concerts frequently. An otoscopic speculum exam reveals no observable abnormalities. Question - What diagnostic test would be most useful in identifying the cause of this patient's hearing loss at this time? Answer Choices 1 Weber test 2 Cochlear biopsy 3 Facial X-rays 4 CT scan of the head 5 Complete blood count
Correct Answer: Weber test - This patient's presentation is most consistent with sensorineural hearing loss due to exposure to loud noise. Tuning forks are useful in differentiating conductive from sensorineural hearing losses. In this patient, the Weber test is expected to reveal a sound that lateralizes to the good or unaffected ear, as cochlear nerve damage impairs transmission to the affected ear. The following, confirmatory Rinne test will demonstrate air conduction greater than bone conduction, the normal pattern. The normal pattern prevails since the cochlear nerve is less able to transmit impulses regardless of how the vibrations reach the cochlea. Collectively, the Weber and Rinne tuning fork tests and a fistula test using pneumatic speculum must be performed. Audiometry is important to perform and includes pure-tone, speech tests, and immittance (tympanometry and acoustic reflex) tests. A cochlear biopsy is invasive and not routinely performed. Facial X-rays have no role in the workup of hearing loss. According to 2012 guidelines from the American Academy of Otolaryngology-Head and Neck Surgery Foundation, computed tomography scans are not helpful and expose the patient to ionizing radiation. CT scans are therefore not recommended in the initial evaluation of patients with presumptive sensorineural hearing loss. Routine, non-targeted laboratory testing is not recommended.
A 12-year-old girl is diagnosed using a quick antigen test as having a pharyngeal infection due to Streptococcus pyogenes. She also presents with a rash on the upper part of the chest and trunk. Question - This rash is caused by what product of Streptococcus pyogenes? Answer Choices 1 Exfoliative toxin 2 Elastase 3 Enterotoxin 4 Erythrogenic toxin 5 Streptolysin
Erythrogenic toxin - Erythrogenic toxin (streptococcal pyrogenic exotoxin) is produced by Streptococcus pyogenes. The toxin is responsible for the rash of scarlet fever. The toxin has been shown to exhibit pyrogenicity and cytotoxicity. It usually appears at the second day of infection, on the upper part of the chest, spreading to the rest of the trunk out towards the rest of the body, sparing the palms, soles, and face. Exfoliative toxin, produced by Staphylococcus aureus, is responsible for scalded skin syndrome (SSS), where the patient appears to have acquired a burn of the skin. There is extensive scaling and flaking desquamation of the epidermis. The syndrome is especially common in infants and small children. Elastase is an extracellular protease produced by Pseudomonas aeruginosa. The protease is associated with the organism's virulence due to tissue destruction and bacterial invasion. Elastase is necrotizing to the skin, cornea, and lung; it is capable of producing hemorrhage. Enterotoxin is produced by various bacteria. In Clostridium perfringens, the production of this toxin causes the symptoms of food poisoning. Clostridium perfringens type A is most associated with this toxin production. Enterotoxin is thought to act as a superantigen, which causes a massive release of inflammatory mediators and induces a calcium ion-dependent breakdown of permeability. Streptolysin is a streptococcal hemolytic exotoxin. There are two types produced by Streptococcus pyogenes: streptolysin O (SLO), which is oxygen-labile, and streptolysin S (SLS), which is oxygen-stable. The production of this hemolytic exotoxin causes the infected host immune response to produce anti-streptolysin O, which can be used to detect Streptococcus pyogenes infections.
A 9-year-old girl presents with a sore throat. The parents state that she began a fever a few days ago, reporting that her throat hurt. On physical exam, you note a red throat, a red and beefy tongue, tonsillar exudates, and swollen anterior cervical lymph nodes. The parents report a history of a severe anaphylactic reaction to penicillin. Question - What antibiotic would treat this infection while minimizing the risk of invoking an allergic reaction? Answer Choices 1 Augmentin 2 Cephalexin (Keflex) 3 Ciprofloxacin 4 Mupirocin (Bactroban) 5 Erythromycin
Erythromycin Explanation - The clinical picture is suggestive of Group A streptococcal pharyngitis. Penicillins are the first-line antibiotics in the treatment of strep pharyngitis. Since the patient has a history of anaphylaxis with penicillin and there is no evidence of skin testing, the guidelines do not recommend the use of a cephalosporin. Instead, these patients should receive a macrolide such as erythromycin or azithromycin as an effective alternative. Augmentin contains amoxicillin, a member of the penicillin family with allergic cross-reactivity; it is contraindicated in patients with a severe allergy to penicillin. Cephalexin, a cephalosporin, can be used to treat strep throat, but approximately 2% of patients who are allergic to penicillin are also allergic to cephalosporins. Ciprofloxacin is effective against Gram-negative organisms. Since streptococcal species are Gram-positive, it would not be an appropriate treatment. Mupirocin is a topical antibiotic and is not indicated for the treatment of strep throat.
A 35-year-old female patient presents with a 24-hour history of purulent drainage and erythema of the right eye. Question - Based on the most likely diagnosis, what is the best treatment for this patient's symptoms? Answer Choices 1 Erythromycin ointment 2 Oral tetracycline 3 Lubricating drops 4 Olopatadine drops 5 Oral acyclovir
Erythromycin ointment Explanation Erythromycin ophthalmic ointment applied 2-4 times daily is the treatment of choice for bacterial conjunctivitis. Other treatment options include trimethoprim-polymyxin B drops. Oral tetracycline is a good treatment choice for chlamydial conjunctivitis, but it is not an appropriate choice for most cases of bacterial conjunctivitis. Lubricating drops are useful for dry eyes but not sufficient to treat bacterial conjunctivitis. Olopatadine is an antihistamine ophthalmic solution used in the treatment of ocular itching associated with allergic conjunctivitis. Acyclovir is an antiviral that is prescribed in cases of herpetic viral conjunctivitis.
During newborn nursery rounds, a young new mother tells you that there is a family history of eye problems that run on her dad's side. She is not sure what the problem is exactly, but many relatives have had to wear glasses. On exam, the infant's eyes appear deviated toward the nose. Corneal light reflex testing confirms the suspected diagnosis. Question - What condition is most likely present? Answer Choices 1 Cataract 2 Glaucoma 3 Retinoblastoma 4 Esotropia 5 Entropion
Esotropia - Congenital esotropia causes the eyes to deviate toward the nose, giving the appearance that they are crossed. The corneal light reflex test (Hirschberg test) can reliably diagnose the condition. The corneal light reflex or small white dot on the front of the eye should be in the same position in each pupil. It is normally just slightly nasal to the center of each pupil. If the position is different in each pupil, then some type of strabismus is present. A prism diopter (PD) is the unit measuring the deflection of light passing through a prism equal to a deflection of 1 cm at a distance of 1 meter. Infantile esotropia characteristically presents as a constant moderate-to-large angle measuring approximately 25-60 PD with alternate fixation. Infants presenting at age 2-4 months with constant esotropia of 40 PD or more are valid candidates for surgical repair. Cataracts present as an opacification or clouding of the lens. Many cataracts in children are acquired within the first several years of life. A normal and equal red reflex in each eye will exclude cataracts. Glaucoma is a common progressive disease characterized by elevated intraocular pressure causing progressive damage to the optic nerve that results in atrophy and blindness. This is due to improper development of the eye's aqueous outflow system. Diffuse corneal haze will obscure the pupil and iris markings, and the symptomatic triad of epiphora (excessive tearing, photophobia, and blepharospasm) is evident. The corneal light reflex is normal in glaucoma. Retinoblastoma usually presents with leukocoria and strabismus. It can also be detected with an abnormal red reflex test. In the absence of a white pupil, this diagnosis is unlikely. Entropion refers to the inversion of the eyelid.
16-year-old girl presents with sore throat and headache. She started feeling poorly when she woke up this morning. Her immunizations are current. Temperature is 102°F. Physical examination is remarkable for erythema in the posterior pharynx and palatal petechiae. She also has tender anterior cervical lymphadenopathy. Question - What organism is the most likely cause of her condition? Answer Choices 1 Group A Streptococcus 2 Corynebacterium diphtheriae 3 Adenovirus 4 Rhinovirus 5 Candida albicans
Group A Streptococcus - Group A Streptococcus is the most common cause of bacterial pharyngitis. The patient has fever, palatal petechiae, and tender anterior cervical adenopathy, all of which are highly suggestive of bacterial pharyngitis. Corynebacterium diphtheriae is an uncommon cause of sore throat in vaccinated children. Patients present with a gray pseudomembrane on examination of the throat. Viral pharyngitis (adenovirus and rhinovirus) can present with erythema and tender anterior cervical adenopathy, but the palatal petechiae are consistent with GAS pharyngitis. Candida albicans of the throat typically presents with thin exudate, but it is not common unless a patient is immunocompromised or has had recent antibiotic treatment.
A 20-year-old woman presents with a 3-day history of fever, sore throat, and enlarged glands in her neck. She denies any cough or runny nose, but she has malaise, body aches, and headaches. She has no other medical problems and does not take any medications. She works in a daycare center; she takes care of children 3-4 years old. On examination, she has a temperature of 101.5°F; pulse is 102/min, and BP 110/70 mm Hg. Oral exam reveals swollen tonsils with plenty of exudates. There is no nasal congestion, and lungs are clear. Cervical lymph nodes are enlarged bilaterally and tender. Abdomen is unremarkable. Throat swab is obtained by the physician. Question - What organism is most likely to be isolated in culture of the swab? Answer Choices 1 Corynebacterium diphtheriae 2 Epstein-Barr virus 3 Haemophilus influenzae 4 Moraxella catarrhalis 5 Group A streptococcus
Group A streptococcus Based solely on the clinical features, the patient is suffering from acute streptococcal pharyngitis, which is characterized by fever, sore throat, exudates, and cervical adenopathy without cough or rhinorrhea. Group A streptococcus is the most likely causative organism among the choices. It is especially seen in children or adults exposed to children in schools, daycare centers, etc. Treatment with penicillin is recommended if rapid antigen test in the office is positive, if the throat culture comes back positive, or if the patient has all four clinical criteria (fever, exudates, cervical lymphadenopathy, and absence of cough). Corynebacterium diphtheriae causes diphtheria and is characterized by pharyngitis, low-grade fever, malaise, and a gray membrane over the pharyngeal area that bleeds on stripping. Throat swab has to be cultured on a specific medium since regular media will not grow this bacterium. Treatment is with penicillin or erythromycin. The Epstein-Barr virus causes infectious mononucleosis that is characterized by fever, respiratory congestion, splenomegaly, hepatomegaly, severe fatigue, as well as anterior and posterior cervical adenopathy. Definitive diagnosis is by noting atypical lymphocytes in the peripheral smear and heterophile antibody test. Serology is needed only in a few cases. Treatment is supportive because antibiotics are useless. Haemophilus influenzae and Moraxella catarrhalis rarely cause pharyngitis. They more commonly cause otitis media or lower respiratory tract infections.
A 35-year-old woman presents with a painful swelling of her left eyelid. On physical exam, there is tenderness to palpation and erythematous swelling present on the lid margin involving the eyelashes. Question - What is the most likely diagnosis? Answer Choices 1 Xanthelasma 2 Chalazion 3 Hordeolum 4 Pinguecula 5 Dacryocystitis
Hordeolum The correct answer is hordeolum (or stye), which is an infection that occurs at the lid margin. It is most often caused by bacteria and is treated with warm compresses and sometimes antibiotic ointment. An internal hordeolum is an infection of the meibomian gland that expands onto the lid conjunctiva. An external hordeolum is typically smaller than an internal one and found on the lid margin. Xanthelasma is a raised yellowish plaque in the skin of the eyelids. It is not painful and is classically associated with high cholesterol. A chalazion is a chronic inflammation of the meibomian gland inside the eyelid, not the lid margin, and is characteristically hard and nontender. Pinguecula is a nodule in the bulbar conjunctiva. Dacryocystitis is an inflammation of the lacrimal sac that leads to pain, swelling, and erythema around the tear sac.
A 12-year-old boy presents with fatigue and jaundice. History obtained from the patient and his mother is negative for recent illness, fever, infectious exposures, medication, alcohol, or drug use. He denies gastrointestinal (GI) symptoms and a history of GI disease. On physical examination, he appears ill; the liver edge is palpable and slightly tender. Skin and sclera are icteric, and there is corneal discoloration. On eye examination using a slit-lamp, you note brown-yellow rings encircling the iris in the rim of the cornea bilaterally. You order a serum ceruloplasmin level, which is reported as low. Question - What is this diagnostic corneal pigmentation known as? Answer Choices 1 Fleischer's rings 2 Kayser-Fleischer rings 3 Rust rings 4 Arcus juvenilis 5 Pinguecula
Kayser-Fleischer rings Kayser-Fleischer rings are the result of the accumulation of copper in the cornea; they are the most unique sign of Wilson's disease. Wilson's disease is an inherited disorder of copper toxicity due to a genetic defect in copper transport. Beginning at birth, copper is not secreted into the bile or incorporated into the copper protein ceruloplasmin, resulting in low serum levels of ceruloplasmin. Symptoms and signs develop age 5-40 as copper accumulates in the liver, brain, cornea, kidney, and reproductive organs. 50% of patients present with hepatitis, 40% present with neurological manifestations (tremor, speech disorders, dysphagia, incoordination), and 5-10% first present with Kayser-Fleischer rings, amenorrhea, miscarriages, or hematuria. Diagnosis is confirmed by Kayser-Fleischer rings on slit lamp examination in the presence of a low serum ceruloplasmin. AST and ALT levels are often elevated; serum copper is low; 24-hour urinary copper excretion is elevated. Treatment is lifelong chelation or oral zinc and a low copper diet. Keratoconus is a bulging of the cornea to form a cone, and the classic sign is Fleischer's rings, which are iron-colored rings surrounding the cone. This progressive bulge is due to a weakness in the cornea and often occurs bilaterally at age 10-20. There are frequent changes in visual acuity, necessitating repeated prescription changes; contacts provide better correction than glasses. Corneal transplant may be necessary if corrective lenses are not adequate. Arcus juvenilis is a gray or white arc around the peripheral cornea similar to arcus senilis in adults. It occurs in younger adults and is often associated with high blood cholesterol. A metallic foreign body lodged in the cornea can quickly result in a single small-diameter rust ring that requires ophthalmologic intervention with a rust ring drill for removal. A pinguecula is a raised, yellowish discoloration on the bulbar conjunctiva at the 3 o'clock or 9 o'clock position of the scleral-corneal junction. It is a benign growth caused by an accumulation of conjunctival tissue that can be the result of chronic actinic irritation.
56-year-old man presents with a history of persistent and progressive unrelenting hoarseness for the last few months. He is a 50 pack-year smoker but quit 1 year ago. Physical examination demonstrated a 2-cm firm non-tender right anterior cervical lymph node. Question - What is the most likely diagnosis? Answer Choices 1 Laryngitis 2 Thyroid cancer 3 Vocal cord nodule 4 Laryngeal cancer 5 Strep pharyngitis
Laryngeal cancer Explanation - The correct answer is laryngeal cancer. Tobacco abuse is a common predisposing factor in laryngeal cancer and affects men more often than women. Persistent hoarseness in this population should cause suspicion of cancer. Many patients with laryngeal cancer present with palpable anterior cervical lymphadenopathy. Acute laryngitis lasts for about 1 week and typically follows a viral infection. Chronic laryngitis—often due to irritants, vocal abuse, or gastroesophageal reflux—does not typically have accompanying non-tender lymphadenopathy. Thyroid cancer may present with anterior cervical lymphadenopathy but is rarely seen with progressive hoarseness. Vocal cord nodules are typically found in patients who overuse their voices and is not related to tobacco abuse. Strep pharyngitis typically causes tender cervical lymphadenopathy unrelated to tobacco abuse and does not cause progressive laryngitis.
A 7-year-old boy presents to the ED with a 2-hour history of epistaxis. He has a history of several nosebleeds, which usually respond to pinching of the nose, but this episode has continued. The patient is known to pick his nose and has had some cold symptoms recently. He did not experience excessive bleeding at circumcision, and there is no family history of bleeding disorders. On exam, he is alert and responsive to questions. Heart rate 120 bpm; respiratory rate 20/min; blood pressure 105/64 mm Hg; oxygen saturation 97% on room air. There is continuous active bleeding from his left nostril. On examination of the nose, no obvious bleeding site can be visualized in the anterior part of the nasal cavity. He receives phenylephrine and nasal packing after the initial evaluation, and his bleeding finally stops. Question - What is the most appropriate next step in his assessment? Answer Choices: 1 MRA of sinuses 2 CT of sinuses 3 Nasopharyngoscopy 4 Chest X-ray 5 Blood type and cross
Nasopharyngoscopy - Nosebleeds are fairly common in children, usually associated with trauma (nose-picking), mucosal friability due to upper respiratory tract infection, and mucosal drying related to environmental conditions. Most episodes are self-limited and require simple measures, such as stopping the bleeding with pressure application (pinching) and comfort care. Recurrent nosebleeds are rarely due to an underlying anatomic or hematologic abnormality. Patients who require further evaluation have: - very frequent nosebleeds - bleeding that is difficult to control or localize - positive family history of bleeding disorder - other signs suggestive of a bleeding disorder In most cases, initial evaluation of patient history and physical examination identifies the source and likely cause of the bleeding. About 90% of nosebleeds occur due to injury to the anterior vascular plexus of Kiesselbach in the anterior nasal septum; this site is usually visible on nasal exam. In this case, the inability to localize the source of the bleeding to the anterior vestibule suggests a more posterior source (bleeding from anterior or posterior ethmoidal or sphenopalatine arteries), which is more difficult to control. Nasopharyngoscopy helps identify the site and direct treatment. If a mass lesion or vascular anomaly is suspected, then CT or MRA may be considered as the next step in evaluation. In this case, chest X-ray would not provide any meaningful information that would help guide management. Blood type and cross is done in patients requiring blood transfusion—not indicated in a patient with stable vital signs.
A 44-year-old man presents for follow-up of poorly controlled type 1 diabetes mellitus that was diagnosed 32 years ago. What change on his funduscopic examination would indicate a need for urgent referral to an ophthalmologist? Answer Choices 1 Blot hemorrhages 2 Cotton wool spots 3 Microaneurysms 4 Neovascularization 5 Flame-shaped hemorrhages
Neovascularization Neovascularization is the hallmark of proliferative diabetic retinopathy. New vessels can appear at the optic nerve and the macula as a result of retinal hypoxia. They are susceptible to rupture, resulting in vitreous hemorrhage, retinal detachment, and blindness. Proliferative retinopathy requires urgent referral to an ophthalmologist and is usually treated with pan-retinal laser photocoagulation. The risk of developing diabetic retinopathy is related to the extent of glycemic control and the duration of diabetes. It is classified as nonproliferative and proliferative. Blot hemorrhages, cotton wool spots, and microaneurysms are indicative of nonproliferative diabetic retinopathy, which is usually seen 10 to 20 years after the onset of diabetes. Nonproliferative retinopathy does not always progress to proliferative retinopathy, but if it becomes extensive, it can result in retinal ischemia, which increases the likelihood of proliferative disease. Flame-shaped hemorrhages are indicative of hypertensive retinopathy.
A 67-year-old woman presents; according to her, her husband says she never listens to anything he says. The patient states that occasionally she has to ask people to repeat themselves when sitting to her right. She denies any dizziness, headaches, or visual disturbances. Her current medication is furosemide. On physical examination, the Weber test reveals lateralization to the left ear. On the left ear, air conduction lasted for 15 seconds and bone conduction lasted 10 seconds. On the right ear, air conduction lasted for 22 seconds and bone conduction lasted 10 seconds. Question - What is the most likely cause of this hearing loss? Answer Choices 1 Cerumen impaction 2 Otosclerosis 3 Ototoxicity 4 Ménière's disease 5 Middle ear effusion
Ototoxicity Ototoxicity secondary to furosemide is the most likely cause. Loop diuretics can cause sensory hearing loss, as evidenced by this patient's physical exam finding of lateralization to the good ear and air conduction slightly longer than bone conduction. The Rinne test should reveal an air-bone conduction ratio of 2:1. Cerumen impaction will cause a conductive hearing loss with the lateralization to the affected ear and a negative Rinne test. Otosclerosis typically will result in conductive hearing loss. Ménière's disease would exhibit any vertigo or tinnitus. Middle ear effusion would cause a conductive hearing loss.
An 18-year-old female college student presents with a 2-day history of severe left ear pain. In the last 5 hours, the pain has become intolerable. Initially, the ear had an intense period of itchiness. Her history is significant for being a member of the college swim team. An examination of the ear canal is remarkable for the presence of edema and redness. A culture swab of the ear canal is performed. The patient is discharged with a course of treatment consisting of polymyxin with a steroid in an acid vehicle, and she is told to return if the symptoms do not subside within the next day. The next day, the microbiology laboratory isolates a gram-negative bacillus; it is oxidase positive and citrate positive. It does not ferment carbohydrates, and it produces a blue-green pigment. Question - What is the cause of this patient's otitis externa? Answer Choices 1 Moraxella catarrhalis 2 Pseudomonas aeruginosa 3 Streptococcus pneumoniae 4 Haemophilus influenzae 5 Bacteroides fragilis
Pseudomonas aeruginosa Pseudomonas aeruginosa is a gram-negative facultative bacillus that grows on MacConkey as a non-lactose fermenter (colonies are not pink). Colonies can appear to have a blue to purple hue. The organism is oxidase positive and citrate positive, and it does not ferment carbohydrates. Pseudomonas aeruginosa infections of the ear are associated with swimming (swimmer's ear, an external otitis). Injury, wet humid conditions, inflammation, and maceration can predispose the external auditory canal to Pseudomonas aeruginosa. Moraxella catarrhalis is a gram-negative coccus that is relatively plump and occurs in pairs. Moraxella catarrhalis is oxidase positive, catalase positive, non-motile, oxidation and fermentation glucose negative, and nitrate negative. It is a commonly isolated pathogen in pediatric patients and immunocompromised/debilitated adults. Moraxella catarrhalis plays an important role in otitis media and sinusitis, as well as nosocomial pneumonia. Streptococcus pneumoniae is a gram-positive coccus that occurs in pairs and is lancet-shaped. The organism is catalase negative and alpha-hemolytic on blood agar, and it grows best at 35°C with a 5% CO2 atmosphere. Streptococcus pneumoniae is the number one cause of acute otitis media. The organism can cause a variety of other infections, including bacteremia and meningitis. Haemophilus influenzae is a gram-negative coccobacillus that is small and light staining. It will only grow on chocolate agar due to the organism requiring the presence of growth factors hemin and NAD for growth. Haemophilus influenzae is the second most common cause of otitis media, most commonly occurring between 6 months and 5 years. The organism can cause a variety of other infections, most importantly meningitis in children as well as sinusitis. Bacteroides fragilis is a gram-negative anaerobic bacillus. Biochemical reactions of significance are growth at 20% bile, indole positive, and resistance to kanamycin/vancomycin/colistin. It can be a cause of chronic otitis media infections and is frequently found concomitantly with other organisms such as Streptococci, Staphylococci, and Pseudomonas aeruginosa.
A 30-year-old male patient presents with recurrent vertigo. He gives a history of attacks when rolling over in bed that last less than 1 minute. He does not have headache, hearing loss, tinnitus, or vomiting. On examination, external auditory canals are unremarkable. Dix-Hallpike maneuver produces nystagmus. Hearing tests are within reference ranges. Question - What is most likely to be useful in treating the patient's condition? Answer Choices 1 Hydrochlorothiazide 2 Methylprednisolone 3 Escitalopram 4 Repositioning maneuvers 5 Scopolamine patch
Repositioning maneuvers - This patient's presentation is consistent with benign paroxysmal positional vertigo caused by calcium debris (canalithiasis) dislodged within the semicircular canal. Most patients present with sudden onset of vertigo often with movement of the head, such as sitting up or laying down when moving, lasting <1 minute. Associated symptoms include nausea and vomiting, but hearing is intact and no other neurologic symptoms are present. Nystagmus with Dix-Hallpike maneuver aids in diagnosis. The most effective treatment is repositioning maneuvers (Epley maneuver) to realign the crystals. Medications are not helpful in the treatment of brief episodes of vertigo and are reserved for cases with higher frequencies of vertigo. These are only helpful in the treatment of symptoms, not the underlying cause. Hydrochlorothiazide is a type of diuretic used in cases of vertigo in Ménière's disease caused by endolymphatic hydrops. This patient does not have symptoms of Ménière's disease, which usually include hearing loss, vertigo, and tinnitus. Methylprednisolone is used to treat vertigo caused by vestibular neuritis associated with instability, nausea, vomiting, vertigo with nystagmus, but patients typically have a history of upper respiratory infections. Escitalopram is a selective serotonin reuptake inhibitor (SSRI) used to treat vertigo in psychosomatic disorders like major depression, anxiety, and panic disorders. Scopolamine is an anticholinergic medication used to treat vertigo caused by motion sickness. Motion sickness typically is the presence of nausea, sweating, and malaise in response to externally imposed motion or other stimuli. Scopolamine causes symptomatic relief of motion sickness, but it is not a treatment for vertigo.
A 2-week-old female infant is seen for her newborn well-baby exam after a normal birth and delivery. She has been nursing well, has regained her birth weight and her development appears normal for her age. Physical examination is normal with the exception that ophthalmoscopic evaluation reveals a faint white reflex in her right eye. Question - What is the most likely diagnosis? Answer Choices 1 Retrolental fibroplasia 2 Phakomata 3 Retinitis pigmentosa 4 Retinoschisis 5 Retinoblastoma
Retinoblastoma Retinoblastoma is the most common primary malignant intraocular tumor of childhood. It usually appears quite early in the first 5 years. Leukocoria, a white or Cat's eye reflex in the pupil is the most frequent finding. There may also be strabismus due to vision impairment. Ocular inflammation, intraocular hemorrhage, glaucoma, or heterochromia iridis may be seen. On fundoscopic exam, the tumor may appear as a small to large white mass depending on its stage. Primary treatment includes enucleation, though smaller tumors diagnosed at an earlier stage may be amenable to newer alternative treatments such as cryotherapy and photocoagulation. Though leukocoria may be seen in retrolental fibroplasia or advanced stage of retinopathy of prematurity, it is predominantly a disorder in preterm, low birth weight infants who received supplemental oxygen in the newborn period. These infants are susceptible due to the immaturity and subsequent damage of developing retinal vasculature. If the retina goes through various stages to ischemia and neovascularization, leukocoria may be seen representing retinal detachment and a subsequent membrane formation. Phakomata are retinal findings hallmarking hamartomatous disorders such as tuberous sclerosis. The distinctive ocular lesion is a yellowish multinodular cystic lesion arising from the retina or disc. Similar lesions can occur in neurofibromatosis. Retinitis pigmentosa is a progressive degeneration of the retina characterized by pigmentary changes, arteriolar attenuation, some degree of optic atrophy and progressively deteriorating visual impairment. Granularity or mottling of the retinal pigment pattern or distinctive focal pigment aggregates can be seen via fundoscopy. Retinoschisis is a congenital disorder involving splitting of the retina into an inner and outer layer. Usually, good vision is maintained. An elevation of the inner layer of the retina can be seen.
A 42-year-old man presents with a 3-day history of "not being able to hear in my right ear." He is otherwise healthy and is not taking any medications. There is no history of trauma. On physical exam, the whisper test is decreased on his right, the Weber test lateralizes to the right ear, and the Rinne test is as follows: right ear bone conduction is greater (lasts longer) than air conduction; left ear air conduction lasts longer than bone conduction. Question - What is the most likely diagnosis? Answer Choices 1 Right ear sensorineural hearing loss, possibly due to Ménière's disease 2 Left ear sensorineural hearing loss, possibly due to Ménière's disease 3 Left ear conductive hearing loss, possibly due to middle ear disease 4 Right ear sensorineural hearing loss, possibly due to acoustic neuroma 5 Right ear conductive hearing loss, possibly due to cerumen impaction
Right ear conductive hearing loss, possibly due to cerumen impaction - Hearing loss can be recognized at the bedside as either sensorineural or conductive through the Rinne and Weber tests. In this case, the Rinne test bone conduction (BC) lasts longer than air conduction (AC) in the right ear (affected ear) and the Weber test lateralizes to the right ear (affected ear). AC normally lasts longer than BC because of the amplifying effects of the eardrum and middle ear. If BC is longer than AC, the patient is likely to have conductive deafness. Both tests in this patient indicate a conductive hearing loss in the right ear, probably produced by cerumen impaction; there is no apparent evidence of middle-ear disease. In sensorineural hearing loss, both AC and BC are equally diminished. The patient's Rinne and Weber test results indicate conductive hearing loss confined to the right ear, not sensorineural hearing loss. There is no evidence of Ménière's disease or acoustic neuroma in this patient.
A newborn presents for a 2-week physical exam. On exam, irregular, white plaques on the buccal mucosa and palate are noted. Upon trying to remove the plaques, there is an underlying erythematous base. Question - What is the most likely diagnosis? Answer Choices 1 Thrush 2 Epstein pearls 3 Bohn's nodules 4 Milk residue 5 Aphthous ulcers
Thrush - Thrush (oral candidiasis) appears as white plaques anywhere in the mouth, and it is not uncommon in neonates who have had exposure to Candida albicans during birth. Removal of the plaques may or may not reveal an erythematous base, but bleeding is uncharacteristic. Aphthous ulcers (canker sores) are painful ulcerations that occur on the oral mucosa. They begin as erythematous indurated papules that erode to form necrotic ulcers with an erythematous halo. Treatment consists of pain relief; the ulcers will heal spontaneously. Bohn's nodules are small cystic lesions located on the buccal and lingual aspects of the maxillary and mandibular ridges in neonates. They are remnants of mucous gland tissue and disappear shortly after birth. Epstein pearls are small white keratinized lesions found on either side of the mid-palatine raphe. They are epithelial tissue trapped during fetal growth that usually disappear shortly after birth. Milk residue may resemble the lesions of thrush, but it easily washes away with flushing, and it is unlikely to have an erythematous base.
A 48-year-old Caucasian man presents with acute onset of blurring of vision and severe pain in the left eye that began 30 minutes ago. He notes seeing halos with his left eye. He is also experiencing nausea and vomiting. These symptoms started at the same time as the pain. The patient reports that he was relaxing on his porch when the pain started. His temperature is 36.9°C, pulse 90/min, BP 130/90 mm Hg, and respirations 20/min. Physical examination reveals a shallow anterior chamber, a hazy cornea, a fixed, moderately dilated pupil, and ciliary injection. Question - What would be the next step in the management of this patient? Answer Choices 1 Lumbar puncture 2 X-ray to rule out a foreign body 3 Topical atropine to the eye to facilitate ophthalmoscopy 4 Tonometry 5 Discharge with topical antibiotic drops for the eye
Tonometry The history and physical examination of this patient are suggestive of acute angle-closure glaucoma, which can be easily confirmed by measuring the intraocular pressure using a tonometer (e.g., the Schiötz tonometer). Acute angle-closure glaucoma develops in individuals with pre-existing anatomic narrowing of the anterior chamber, which is seen mainly in far-sighted people. The condition usually develops in the twilight hours, which is when the pupil is dilated in response to the low level of illumination. It may also occur with pupillary dilation for ophthalmoscopy, so topical atropine would be contraindicated. Acute angle-closure glaucoma is an ophthalmologic emergency. Treatment involves immediate lowering of the intraocular pressure via systemic acetazolamide to decrease the production of aqueous humor, which should be supplemented with topical hyperosmotic agents and topical beta blockers. Topical pilocarpine is then used to cause miosis. Once the intraocular pressure is under control, a peripheral iridectomy can be done to prevent against future recurrences.
A 6-year-old girl is brought in by her mother and presents with sudden localized swelling of her left upper eyelid at the lid margin. Mom states this has never happened to the girl before. She denies discharge, fever, or trauma. There is mild pain on to palpation. The remainder of the eye exam is within normal limits. Question - Based on the most likely diagnosis, what treatment should be given? Answer Choices 1 Warm compresses 2 Surgical incision and drainage 3 Systemic antibiotics 4 Topical antiviral 5 Hydrocortisone ointment
Warm compresses - The usual causative agent of hordeolum/stye is Staphylococcus aureus. Treatment involves warm compresses and frequently a topical ophthalmic antibiotic. Occasionally, surgical incision and drainage are required. There is no need for a systemic antibiotic for a minor infection, and the causative agent does not appear to be allergic or viral. - A stye can be confused with a chalazion, which is a granulomatous inflammation of a meibomian gland characterized by a non-tender nodule. The lesion tends to be chronic, and it shows no sign of acute inflammation.
A 52-year-old man presents with concerns over hearing changes. He has noticed a decreased ability to hear sounds for the past few months; he tested it at home by covering each ear, and he now thinks there is a hearing loss in only the left side. He also hears a ringing sound all the time. He denies occupational exposure to loud noises. He denies head trauma, headaches, and prior ear problems. His wife thinks this is just normal age-related hearing loss. His review of systems is negative for other neurological symptoms. Past medical history is unremarkable; he has no known medical conditions. He takes no medications. He has no allergies, and he has not had any surgeries. He denies alcohol, tobacco, and recreational drug use. On physical exam, his vitals are normal. His HEENT exam is significant only for decreased auditory acuity and Weber test lateralizing to the right. Audiometry confirms a sensorineural hearing loss on the left. An MRI shows a well-delineated intracranial mass. Further investigation reveals the origin of cells is from Schwann cells. Question After completing treatment for his current condition, what is the best approach for health maintenance? Answer Choices 1 Full
Yearly head imaging - This presents with a vestibular schwannoma (acoustic neuroma) affecting his vestibulocochlear nerve (cranial nerve VIII). This is one of the more common benign head and neck neoplasms. A common presentation is unilateral hearing loss and tinnitus. Treatment is typically surgical removal, or possibly radiation therapy. After completing therapy, yearly head imaging is recommended to monitor for recurrence. Full body bone scans every 2 years are not recommended for vestibular schwannomas; they are not likely to metastasize to bone. Prophylactic antibiotics have no role in the original formation or the recurrence of schwannomas. Some other benign neoplasms in the ear, such as cholesteatomas, are associated with chronic ear infections, but antibiotics are still not helpful after treatment of those growths. Serum blood tests of inflammatory markers every 6 months are not helpful for schwannoma monitoring. Schwannomas are not considered an inflammatory disease and typical serum markers (erythrocyte sedimentation rate and C-reactive protein) may be entirely normal in affected individuals. Testing only if the patient reports return of symptoms is not appropriate because of the slow-growing nature of schwannomas. Patients with schwannomas may adapt to slow hearing loss or balance problems (if the vestibular portion of the nerve is affected) and may easily miss these symptoms for years.
What do we use tonometry for?
measuring the intraocular pressure using a tonometer (e.g., the Schiötz tonometer) The history and physical examination of this patient are suggestive of acute angle-closure glaucoma, which can be easily confirmed by measuring the intraocular pressure using a tonometer (e.g., the Schiötz tonometer).